0% found this document useful (0 votes)
43 views39 pages

STEP 2 Sample

Uploaded by

tsartsova
Copyright
© © All Rights Reserved
We take content rights seriously. If you suspect this is your content, claim it here.
Available Formats
Download as PDF, TXT or read online on Scribd
0% found this document useful (0 votes)
43 views39 pages

STEP 2 Sample

Uploaded by

tsartsova
Copyright
© © All Rights Reserved
We take content rights seriously. If you suspect this is your content, claim it here.
Available Formats
Download as PDF, TXT or read online on Scribd
You are on page 1/ 39

UWORLD

USMLE STEP 2
QUESTIONS
MEDICINE IV

SEPTEMBER 2024
Table Of Contents

Medicine ....................................................................................................................... 3

Nervous System (Miscellaneous) ............................................................................... 3

Nervous System (Neurodegenerative disorders and dementias) ............................. 40

Nervous System (Normal structure and function of the nervous system) ................. 91

Nervous System (Seizures and epilepsy)................................................................. 97

Nervous System (Sleep disorders)......................................................................... 110

Nervous System (Spinal cord disorders) ................................................................ 115

Nervous System (Traumatic brain injuries) ............................................................ 146

Nervous System (Tumors of the nervous system) .................................................. 160

Ophthalmology (Disorders of the eye and associated structures) .......................... 170

Poisoning & Environmental Exposure (Environmental exposure)........................... 215

Poisoning & Environmental Exposure (Toxicology) ................................................ 237

Psychiatric/Behavioral & Substance Use Disorder (Miscellaneous) ....................... 269

Psychiatric/Behavioral & Substance Use Disorder (Mood disorders) ..................... 271

Psychiatric/Behavioral & Substance Use Disorder (Neurodevelopmental and


neurocognitive disorders) ....................................................................................... 274

Psychiatric/Behavioral & Substance Use Disorder (Somatoform disorders and sleep


disorders) ............................................................................................................... 276

Psychiatric/Behavioral & Substance Use Disorder (Substance use disorders) ....... 282

Pulmonary & Critical Care (Cancer and pulmonary/mediastinal masses) ............... 285

Pulmonary & Critical Care (Congenital and developmental anomalies).................. 307

Pulmonary & Critical Care (Critical care and trauma medicine) .............................. 309
UWORLD USMLE STEP 2 QBANK - MEDICINE - NERVOUS SYSTEM (MISCELLANEOUS) - September 2024

Pulmonary & Critical Care (Interstitial pulmonary and other systemic disorders) .... 346

.............................................................................................................................. 362

Pulmonary & Critical Care (Miscellaneous) ............................................................ 368

Pulmonary & Critical Care (Normal pulmonary structure and function) .................. 381

Pulmonary & Critical Care (Obstructive and restrictive lung disease) ..................... 385

Pulmonary & Critical Care (Pulmonary infections).................................................. 441

Pulmonary & Critical Care (Pulmonary vascular and cardiopulmonary disease) .... 481

Pulmonary & Critical Care (Sleep disorders) .......................................................... 515

Renal, Urinary Systems & Electrolytes (Acute kidney injury) .................................. 518

imedicaldoctor.net 2
UWORLD USMLE STEP 2 QBANK - MEDICINE - NERVOUS SYSTEM (MISCELLANEOUS) - September 2024

Medicine

Nervous System (Miscellaneous)


1. A 60-year-old man comes to the office for evaluation of gait imbalance, which he has had for the past 3
days. The patient was hospitalized 2 weeks ago for a complicated enterococcal diabetic infection of the left
foot. He was prescribed ampicillin and gentamicin, which he has continued via home intravenous antibiotic
therapy. The patient subsequently developed a constant sensation of objects moving around in his visual field
when looking in any direction, which has caused him to feel unsteady. He has no associated nausea or
vomiting. The patient has a history of poorly controlled type 2 diabetes mellitus and hypertension. His other
medications include lisinopril and basal-bolus insulin therapy. Temperature is 36.7 C (98 F), blood pressure is
140/96 mm Hg, pulse is 80/min, and respirations are 16/min. BMI is 34 kg/m2. The left foot wound is healing
without discharge or surrounding erythema. Neurologic examination shows 5/5 muscle strength and 2+ reflexes
in all extremities. Hearing is decreased in the right ear on whisper test. Rapid head movement away from a
fixed target causes eye movement away from the target followed by a horizontal saccade back toward it. Which
of the following is the most likely cause of this patient's current symptoms? (QID: 2837)
A. Benign paroxysmal positional vertigo B. Cerebellar infarction
C. Diabetic autonomic dysfunction D. Drug toxicity
E. Ménière disease
F. Vertebrobasilar insufficiency

Correct Answer is D (57% answered correctly)


This patient who was recently prescribed gentamicin now has new-onset hearing loss, imbalance, and a
sensation of objects moving (oscillopsia). This presentation is consistent with aminoglycoside
ototoxicity. The risk of aminoglycoside ototoxicity is increased in patients with bacteremia, hepatic or renal
dysfunction, or when used in combination with another potentially ototoxic medication.
Aminoglycoside ototoxicity can damage hair cells in the cochlea (causing hearing loss) and/or the vestibular
system (causing imbalance). Therefore, patients have varying clinical features, including the following:
• Bilateral, but not necessarily symmetric, hearing loss (as evidenced by this patient's right > left screening
whisper test). Patients with a positive whisper test require a formal audiogram.
• Imbalance and oscillopsia because bilateral vestibular systems are affected (rather than true vertigo,
which generally occurs when 1 side is affected).
• A positive head thrust test (ie, inability to maintain visual fixation during forced, rapid head
movement). This test evaluates the vestibuloocular reflex, which is affected by peripheral but not central
vestibulopathies, as seen in this patient.
Management of patients with aminoglycoside toxicity requires immediate discontinuation of the medication to
prevent further damage.
(Choice A) In contrast to this patient, those with benign paroxysmal positional vertigo (BPPV) have brief (<1
minute), episodic vertigo stimulated by specific head movements (eg, rolling over in bed, looking up). In addition,
BPPV is not associated with hearing loss and is confirmed with the Dix-Hallpike maneuver (vertigo and
nystagmus on quickly lying back into a supine position with the head rotated 45 degrees) rather than the head
thrust test.
(Choices B and F) This patient has risk factors for cerebellar stroke and vertebrobasilar insufficiency, which
can cause gait imbalance. However, these cause central vertigo and therefore would have a preserved head

imedicaldoctor.net 3
UWORLD USMLE STEP 2 QBANK - MEDICINE - NERVOUS SYSTEM (MISCELLANEOUS) - September 2024

thrust and no hearing loss.


(Choice C) Poorly controlled diabetes can cause autonomic dysfunction (eg, gait imbalance, "dizziness" while
standing) due to postural hypotension. However, it does not cause a positive head thrust test, oscillopsia, or
hearing loss, making this diagnosis unlikely.
(Choice E) Ménière disease is an inner ear disorder characterized by increased endolymph volume and/or
pressure (endolymphatic hydrops). It presents with discrete episodes of spinning vertigo (vs constant sensation
in this patient) accompanied by hearing loss (which can be progressive) as well as tinnitus or aural fullness (not
seen in this patient). In addition, the timeline of symptoms shortly after aminoglycoside therapy makes drug
toxicity much more likely.
Educational objective:
Aminoglycosides can be ototoxic to both the cochlea (resulting in sensorineural hearing loss) and the vestibular
system (resulting in imbalance).

2. A 23-year-old woman with a history of anorexia nervosa is evaluated due to acute-onset confusion while
hospitalized. Over the past 6 months, she has been severely restricting her caloric intake and has lost 7 kg (15.4
lb). On admission, she was tachycardic and orthostatic, and her BMI was 15 kg/m2. The patient has been
receiving intravenous hydration and parenteral nutrition. Today, her family notes that she seems newly confused
and unsteady when walking. Temperature is 36.7 C (98 F), blood pressure is 110/70 mm Hg, and pulse is
86/min. She is not oriented to time or place. Bilateral pupils are equal and reactive. Her lateral gaze is restricted
on both sides and evokes a horizontal nystagmus. There is no nuchal rigidity or motor weakness. Bilateral ankle
reflexes are diminished. The patient walks slowly with short and wide-based steps. Which of the following is
the best next step in management of this patient? (QID: 3082)
A. Intramuscular cobalamin B. Intravenous acyclovir
C. Parenteral phosphate D. Systemic glucocorticoids
E. Thiamine supplement
F. Thrombolytic therapy

Correct Answer is E (69% answered correctly)


Wernicke encephalopathy
• Chronic alcohol use (most common)
Associated
• Malnutrition (eg, anorexia nervosa)
conditions
• Hyperemesis gravidarum
Pathophysiology • Thiamine deficiency
• Encephalopathy
• Oculomotor dysfunction (eg, horizontal nystagmus & bilateral abducens
Clinical features
palsy)
• Postural & gait ataxia
Treatment • Intravenous thiamine followed by glucose infusion

imedicaldoctor.net 4
UWORLD USMLE STEP 2 QBANK - MEDICINE - NERVOUS SYSTEM (MISCELLANEOUS) - September 2024

This patient being treated for anorexia nervosa likely has developed Wernicke encephalopathy (WE), a
disorder characterized by:
• Encephalopathy (eg, confusion)
• Oculomotor dysfunction (eg, bilateral abducens palsy, horizontal nystagmus)
• Gait ataxia (eg, wide-based gait).
WE occurs in patients with long-term thiamine (vitamin B1) deficiency due to poor dietary intake (eg, anorexia,
chronic alcohol use), impaired metabolism, or poor absorption.
WE is diagnosed based on the triad of clinical findings; no laboratory or radiologic studies are necessary. When
WE is suspected, intravenous thiamine should be administered immediately. Because the body's requirements
for thiamine (a cofactor for many enzymes) increases with high metabolic rate or glucose intake, the
administration of glucose before thiamine (as likely occurred in this patient receiving intravenous hydration) can
induce or worsen the condition, which can lead to coma or death.
(Choice A) Cobalamin (vitamin B12) deficiency can cause neurologic deficits, including impaired vibratory and
positional sense, gait ataxia, and megaloblastic anemia. Mental status changes can also be seen, but
oculomotor symptoms would be unusual.
(Choice B) Acyclovir is used to treat herpes simplex encephalitis, a central nervous system (CNS) viral infection
that can cause various neurological effects (eg, seizure, altered mental status); however, fever and headache
are typically present whereas nystagmus and ataxia are less likely.
(Choice C) Refeeding syndrome (RS), sometimes seen in profoundly malnourished patients receiving
aggressive nutritional supplementation, is characterized by hypophosphatemia (requiring parenteral phosphate)
and volume overload due to shifting nutrients and fluids, respectively. Although RS is sometimes associated
with thiamine deficiency, this patient has no other manifestations of RS (eg, edema, heart failure,
seizures). Oculomotor findings and ataxia are atypical.
(Choice D) Systemic glucocorticoids can be used to treat multiple sclerosis, an autoimmune inflammatory
demyelinating disorder of the CNS typically causing neurologic deficits disseminated in space and time (eg,
sensory loss/paresthesias of the extremities, dizziness, optic neuritis) in women age 15-50. It does not usually
begin with acute confusion.
(Choice F) Thrombolytic therapy can be used for an acute stroke causing neurologic deficits. An infarction of
the medial vermis can result in vertigo and nystagmus, whereas a lateral cerebellar infarction can cause
dizziness, ataxia, and weakness. However, an infarct would be unusual in a young woman without other risk
factors.
Educational objective:
Thiamine deficiency can cause Wernicke encephalopathy, which is characterized by encephalopathy,
oculomotor dysfunction, and gait ataxia. This is generally seen in malnourished patients (eg, anorexia, chronic
alcohol use) and may be induced iatrogenically by the administration of glucose without thiamine.

3. A 62-year-old man comes to the office due to tremors. The patient first noticed them several years ago, but
they have progressively worsened and now cause difficulty with daily activities such as drinking tea and buttoning
clothing. His wife reports that he also has a mild head tremor. His father had similar symptoms that worsened
as he advanced in age. The patient has a history of osteoarthritis, hypertension, and gastroesophageal reflux
disease. He does not drink alcohol. Blood pressure is 144/80 mm Hg and pulse is 82/min. Physical examination
shows a mild tremor of the head and a tremor of the bilateral upper extremities when the arms are extended. The
tremor worsens during finger-to-nose testing. Muscle tone, deep tendon reflexes, limb coordination, and gait are
normal. Serum electrolytes, blood glucose, and TSH are normal. Which of the following is the most appropriate
next step in management? (QID: 4179)
A. MRI of the brain B. No additional testing
C. Plasma metanephrines D. Serum ceruloplasmin levels
E. Striatal dopamine transporter imaging

imedicaldoctor.net 5
UWORLD USMLE STEP 2 QBANK - MEDICINE - NERVOUS SYSTEM (MISCELLANEOUS) - September 2024

Correct Answer is B (75% answered correctly)


Essential tremor
• Family history often present
Epidemiology
• Up to 5% of population affected
• Action tremor (eg, hands outstretched)
Clinical • Usually bilateral
features • Hands > arms > head >> legs
• Worsens with stress, improves with alcohol
• Beta blockers: propranolol
Treatment
• Anticonvulsants: primidone
This patient has a progressively worsening tremor of the distal upper extremities that becomes more pronounced
with extension of his arms. It worsens even more significantly at the end of an intended action (finger-to-nose
test). This patient also has a head tremor and a positive family history. In the absence of other neurologic signs
or symptoms, these findings are most consistent with essential tremor (ET).
Although its features are highly variable, ET most often affects the wrists and hands, especially when the arms
are extended. It worsens with movement (ie, action tremor) and improves, but may not completely abate, at
rest. Although ET is not typically unilateral, it may be asymmetric. It can also affect the head and voice but
generally spares the trunk and legs. ET is slowly progressive and often hereditary; it can be passed in an
autosomal dominant fashion. It classically improves with small amounts of alcohol. Features of parkinsonism
(eg, rigidity, gait disturbances) are absent.
In most cases, the diagnosis of ET is clinical; further diagnostic testing and neuroimaging are not indicated
(Choice A). In rare cases when it is difficult to distinguish between ET and Parkinson disease (eg, in those with
bradykinesia or rigidity), striatal dopamine transporter imaging, which is normal in patients with ET, can be
performed (Choice E). Beta blockers such as propranolol are first-line therapy; patients may also receive an
anticonvulsant such as primidone alone or in combination.
(Choice C) Plasma metanephrines are used to diagnose pheochromocytoma. Although some patients have
paroxysmal tremors associated with hyperadrenergic spells, this disease classically presents with episodic
headaches, sweating, and tachycardia with hypertension.
(Choice D) Serum ceruloplasmin is used to diagnose Wilson disease, which is characterized by abnormal
copper deposition in tissues such as the liver, basal ganglia, and cornea. It can present with neurologic findings,
including tremor. However, patients typically have additional neurologic findings (eg, parkinsonism, dysarthria),
psychiatric symptoms, and liver disease (eg, cirrhosis).
Educational objective:
Essential tremor most often presents as a tremor of the hands that is suppressed at rest, exacerbated by
outstretched arms, and more pronounced at the end of goal-directed movements. It is often hereditary and can
be associated with a head tremor, but it is not associated with other neurologic symptoms. The diagnosis is
clinical.

4. A 78-year-old woman is brought to the emergency department due to insomnia and frequent episodes of
agitation over the past 2 days. The patient was confused yet calm on arrival; but now she is yelling loudly and
trying to pull out her intravenous lines, take off her clothes, and run away while being examined. She is a nursing
home resident. Her recent medical history includes severe memory loss. The patient's chronic medical issues
include hypertension, type 2 diabetes mellitus, peptic ulcer disease, and chronic pyelonephritis. Her temperature
is 37.2 C (99 F), blood pressure is 162/96 mm Hg, and pulse is 95/min and regular. Pulse oximetry shows 96%
oxygen saturation on room air. ECG shows normal sinus rhythm, left ventricular hypertrophy, a normal QTc
interval, and no acute ischemic changes. Laboratory studies are as follows:

imedicaldoctor.net 6
UWORLD USMLE STEP 2 QBANK - MEDICINE - NERVOUS SYSTEM (MISCELLANEOUS) - September 2024

Hematocrit 41%
Leukocytes 12,000/mm3
Platelets 160,000/mm3
Sodium 137 mEq/L
Potassium 4.8 mEq/L
Chloride 95 mEq/L
Creatinine 1.2 mg/dL
Blood urea nitrogen 25 mg/dL
Urinalysis shows trace protein, numerous leukocytes, and occasional erythrocytes. Which of the following is the
best initial treatment for this patient? (QID: 4622)
A. Amitriptyline B. Clopidogrel
C. Haloperidol D. Lorazepam
E. Memantine

Correct Answer is C (67% answered correctly)


Causes of delirium
• Dementia
• Parkinson disease
Predisposing
• Prior stroke
risk factors
• Advanced age
• Sensory impairment
• Drugs (eg, narcotics, sedatives, antihistamines, muscle relaxers,
polypharmacy)
• Infections (eg, pneumonia, urinary tract infection, meningitis)
Precipitating • Electrolyte disturbances (eg, hyponatremia, hypercalcemia)
factors • Metabolic derangements (eg, volume depletion, vitamin B12 deficiency,
hyperglycemia)
• Systemic illnesses (eg, congestive heart failure, hepatic failure, malignancy)
• CNS conditions (eg, seizure, stroke, head injury, subdural hematoma)
Delirium (waxing and waning alteration in consciousness) in hospitalized patients is usually due to toxic-
metabolic or infectious etiologies. Advanced age and dementia are also important risk factors. A workup for the
cause of delirium (review of medication list, blood work, urinalysis, possible imaging) is needed (Table). If an
infection is identified, treatment should be initiated as soon as possible. In this patient with underlying dementia,
delirium may be due to a urinary tract infection given the pyuria seen on urinalysis.
Regardless of the etiology of her dementia and delirium, this patient has severe agitation, which may
compromise her safety and interfere with medical management (eg, difficulty with intravenous fluid or antibiotic
administration, radiologic testing). In such cases, low-dose haloperidol can be considered. Atypical
antipsychotics (quetiapine, risperidone) may also be used. Usage should be limited to the shortest possible
duration, as prolonged use of antipsychotics can increase mortality in the elderly. Typical antipsychotics should
not be used in patients with Lewy body dementia, who may exhibit neuroleptic hypersensitivity (severe
parkinsonism and impaired consciousness with neuroleptic administration).

imedicaldoctor.net 7
UWORLD USMLE STEP 2 QBANK - MEDICINE - NERVOUS SYSTEM (MISCELLANEOUS) - September 2024

(Choice A) Amitriptyline is a tricyclic antidepressant used to treat depression, sleep disorders, and neuropathic
pain. It has significant anticholinergic side effects. For this reason, amitriptyline usage for elderly patients is
generally discouraged, especially for those with underlying dementia.
(Choice B) Clopidogrel is sometimes used in the management of acute coronary syndrome and ischemic
stroke. It plays no role in treating agitation/delirium.
(Choice D) Lorazepam and other benzodiazepines may be used to treat agitation in young patients. However,
they are typically contraindicated in older patients, who are at increased risk for adverse events (eg, withdrawal,
dependence, motor impairment), may experience worsening agitation (paradoxic effect), and tend to metabolize
benzodiazepines slowly, making their effects very long-lasting.
(Choice E) Memantine is an agent used to treat moderate to severe Alzheimer disease. It works by blocking
the action of glutamate on the N-methyl-D-aspartate (NMDA) receptor. Memantine may improve the cognitive
symptoms associated with this patient's dementia, but it will not help her acute agitation.
Educational objective:
Toxic-metabolic and infectious etiologies are the most common causes of delirium in a hospitalized
patient. Patients with dementia have an increased risk of developing agitated delirium in the hospital. Typical
and atypical antipsychotics are useful for treating acute agitation in elderly patients with
dementia. Benzodiazepines are typically not recommended in this setting.

5. A 76-year-old woman is brought to the emergency department by her son with 2 days of intermittent confusion,
daytime somnolence, and decreased oral intake. Her medical conditions include mild dementia, hypertension,
and type 2 diabetes mellitus. The son says that the patient has been talking to people who are not there and
wandering around the house in the middle of the night. She has no history of recent falls. Medications include
valsartan and metformin. Temperature is 37.2 C (99 F), blood pressure is 100/60 mm Hg, and pulse is 100/min
and regular. The patient is easily distracted and disoriented to time and place. Neck is supple. She can move
all 4 extremities. Cardiopulmonary examination is normal. Abdomen is soft and nontender. Fingerstick glucose
level is 155 mg/dL. Which of the following is the most appropriate next step in management? (QID: 4644)
A. CT scan of the head with contrast B. Electroencephalogram
C. Lumbar puncture D. Serum ammonia level
E. Urinalysis

Correct Answer is E (75% answered correctly)

Mental status changes are commonly due to delirium;


however, delirium is frequently under-recognized. The
Confusion Assessment Method (CAM) improves
delirium identification and differentiates it from other
cognitive disturbances (eg, dementia, depression). The
CAM assesses 4 fundamental features of delirium:
1. acute change (eg, hours to days) and fluctuating
course (eg, intermittent)
2. inattention (eg, easy distractibility)
3. disorganized thinking (eg, confusion)
4. altered consciousness (eg, daytime somnolence)
This patient has all 4 features (delirium diagnosis
formally requires both features 1 and 2 plus either feature
3 or 4). Notably, in patients with preexisting cognitive
deficits (eg, advanced age, dementia), delirium may be the only sign of acute illness (eg, afebrile, no
dysuria). This patient also has hallucinations and sleep-wake reversal, which are common with delirium but not

imedicaldoctor.net 8
UWORLD USMLE STEP 2 QBANK - MEDICINE - NERVOUS SYSTEM (MISCELLANEOUS) - September 2024

essential for diagnosis.


Delirium indicates acute brain failure and should be considered a medical emergency. Common causes of
delirium include infection (eg, cystitis, pneumonia), adverse drug effects, dehydration, and electrolyte/metabolic
disturbances. Initial evaluation includes a history and physical examination, medication review, complete blood
count, metabolic panel, and urinalysis. This will identify most causes of delirium; however, advanced workup
should follow any abnormal clinical findings or negative initial evaluation.
(Choices A and C) CT scan of the head is indicated in patients with altered mental status when history (eg,
trauma, anticoagulation) or physical examination (eg, focal weakness, papilledema) suggests intracranial
pathology. Lumbar puncture is typically used to diagnose meningitis or encephalitis. However, this patient has
no concerning history, focal neurologic findings, fever, or meningismus, so more common delirium causes should
be ruled out first. If initial workup is negative or treatment response is inadequate, neuroimaging and lumbar
puncture should be considered.
(Choice B) Electroencephalogram (EEG) can identify occult seizures as the cause of altered mental
status. Patients with a concerning history (eg, stroke, trauma, critical illness) or inadequate response to delirium
treatment require EEG to rule out nonconvulsive seizures as a cause of delirium.
(Choice D) Ammonia is neurotoxic and elevated levels can cause delirium. Hyperammonemia most commonly
results from acute or chronic liver failure (eg, cirrhosis), which would be evident on initial evaluation (eg, jaundice,
aminotransferase elevation).
Educational objective:
Delirium is common but under-recognized. The Confusion Assessment Method improves delirium identification
using its fundamental features of acute-onset, fluctuation, inattention, disorganized thinking, and altered
consciousness. Initial evaluation is directed at the most common causes, but further assessment should follow
any abnormal findings or negative initial evaluation.

6. A middle-aged woman is found wandering the streets with an abnormal gait. She is brought to the hospital
by police officers. When asked for identification, the patient mumbles incoherently. She is not oriented to time
or place. Her temperature is 36.2 C (97.3 F), blood pressure is 160/100 mm Hg, pulse is 100/min, and
respirations are 18/min. BMI is 17 kg/m2. Head and neck examination shows bitemporal wasting and dry
mucous membranes. The pupils are 3 mm bilaterally and react slowly to light. Her neck is supple, and she
moves all extremities equally. Deep-tendon reflexes are symmetrical bilaterally. Which of the following is the
best initial treatment for this patient? (QID: 4700)
A. Flumazenil B. Haloperidol
C. Labetalol D. Naloxone
E. Thiamine

Correct Answer is E (68% answered correctly)

imedicaldoctor.net 9
UWORLD USMLE STEP 2 QBANK - MEDICINE - NERVOUS SYSTEM (MISCELLANEOUS) - September 2024

Causes of altered mental status


• Prescription drugs (eg, opioids, lithium, antipsychotics)
Drugs/toxins • Drugs of abuse (eg, ethanol, hallucinogens)
• Drug withdrawal (eg, ethanol, benzodiazepines)
• Sepsis, systemic infections
Infections
• Meningitis, encephalitis, brain/epidural abscess
• Electrolyte disturbances
• Hypo-/hyperglycemia
• Endocrine (eg, hypo/hyperthyroid, pituitary, adrenal)
Metabolic
• Hypoxemia, hypercarbia
• Nutritional (eg, thiamine, vitamin B12 deficiency)
• Hepatic or renal failure
• Seizure, head injury
Central nervous system • Hypertensive encephalopathy
• Psychiatric disorders
Altered mental status (AMS), as in this patient, suggests widespread dysfunction of the cerebral cortex and/or
reticular activating system. AMS can be due to a broad spectrum of disorders, including psychiatric illness,
substance intoxication/withdrawal, infection, metabolic derangements (eg, hypoglycemia, hypernatremia),
primary central nervous system disorders (eg, seizures), trauma, or organ dysfunction (respiratory, liver, or renal
failure).
Chronic malnutrition, especially in the setting of alcoholism, can lead to thiamine (vitamin B 1) deficiency with
progression to Wernicke encephalopathy (WE). WE typically presents with the classic triad of encephalopathy,
ocular dysfunction, and gait ataxia. Giving thiamine promptly in suspected WE may prevent further
complications. Thiamine is a cofactor in many enzymes required for energy metabolism, and giving intravenous
fluids containing glucose prior to thiamine can precipitate or worsen WE. Therefore, thiamine should be given
along with or before glucose. When the cause of AMS is not apparent after initial assessment, empiric
treatment for likely causes (eg, WE) is often appropriate.
(Choice A) Flumazenil is a competitive antagonist of the GABA/benzodiazepine receptor and is used to treat
benzodiazepine overdose. Patients typically have slurred speech, ataxia, hypotension, and depressed mental
status. This patient's hypertension makes this unlikely.
(Choice B) Haloperidol is an antipsychotic medication used to treat schizophrenia and acute psychotic
states. However, this patient should have medical causes of altered mental status excluded before receiving
haloperidol.
(Choice C) Labetalol can treat hypertensive encephalopathy, which usually presents with blood pressure
>180/120 mm Hg, headache, nausea, vomiting, and confusion. This patient's moderate blood pressure elevation
is more likely due to the underlying condition and will likely resolve after appropriate treatment.
(Choice D) Naloxone is an opioid antagonist used to treat opioid overdose, which typically presents with pinpoint
pupils and respiratory depression (respirations usually <12/min).
Educational objective:
Wernicke encephalopathy is due to thiamine (vitamin B1) deficiency and is most commonly seen in malnourished
patients with underlying alcoholism. Features include encephalopathy, ocular dysfunction, and gait
ataxia. Patients should be treated empirically with thiamine prior to or along with glucose administration.

7. A 32-year-old man is brought to the emergency department after his coworkers found him confused,
disoriented, and bleeding from the nose. The patient's medical history is significant for an episode of major

imedicaldoctor.net 10
UWORLD USMLE STEP 2 QBANK - MEDICINE - NERVOUS SYSTEM (MISCELLANEOUS) - September 2024

depression. He is currently taking no medication. According to his friends, he was in his normal state of health
this morning when he came to work. He then spent the morning cutting several bushes and trees to clean the
area for a new road construction. Temperature is 42 C (108 F), blood pressure is 110/70 mm Hg, respirations
are 16/min, and pulse is 120/min and regular. BMI is 40 kg/m2. The patient's skin is warm and dry, and his neck
is supple with no stiffness. The pupils are symmetric, midsize, and reactive to light. There is active bleeding
from the right nostril. Deep tendon reflexes are symmetric. No Babinski sign is present. The patient moves all
the extremities but is unable to speak or follow simple commands. Skin on his extremities has several scratch
marks, likely from thorn injuries. Which of the following is the most likely diagnosis? (QID: 4703)
A. Anticholinergic toxicity B. Heat stroke
C. Malignant hyperthermia D. Meningococcal meningitis
E. Salicylate overdose

Correct Answer is B (68% answered correctly)


Exertional heat stroke
• Strenuous activity during hot & humid
weather
• Dehydration, poor acclimatization
Risk factors • Lack of physical fitness, obesity
• Medications: anticholinergics,
antihistamines, phenothiazines, tricyclics,
antipsychotics
• Core temperature >40 C (104 F) with CNS
dysfunction (eg, AMS, seizure)
Clinical • Organ or tissue damage: renal or hepatic
manifestations failure, disseminated intravascular
coagulation, acute respiratory distress
syndrome
• Rapid cooling: ice water immersion
preferred; can consider high-flow cold
water dousing, ice-wet towel rotation,
Management evaporative cooling
• Fluid resuscitation, electrolyte correction
• Management of end-organ complications
• No role for antipyretic therapy
AMS = altered mental status.
This patient with severe hyperthermia, encephalopathy, and epistaxis that developed while working outside has
exertional heat stroke (EHS). EHS is characterized by core temperature ≥40 C (104 F) with CNS dysfunction
(eg, altered mental status). Unlike nonexertional heat stroke, which typically affects individuals at the extremes
of age who are incapable of obtaining adequate fluids and removing themselves from a hot environment, EHS
occurs most commonly in those exposed to hot/humid environments while performing extreme physical activity
(eg, landscaping, as with this patient). Risk factors include obesity, poor physical fitness, lack of acclimatization,
and certain medications/drugs (eg, anticholinergics, alcohol, amphetamines) that impair thermoregulation.
In addition to CNS dysfunction, other common manifestations of EHS include tachycardia, tachypnea, and
flushing. Diaphoresis may or may not be present, depending on fluid status, and hypotension may
occur. Complications include rhabdomyolysis, disseminated intravascular coagulation (DIC) (resulting in

imedicaldoctor.net 11
UWORLD USMLE STEP 2 QBANK - MEDICINE - NERVOUS SYSTEM (MISCELLANEOUS) - September 2024

coagulopathic bleeding, as seen in this patient with epistaxis), and end-organ dysfunction (ie, renal/hepatic
failure). Management includes patient stabilization (eg, airway, breathing, circulation), rapid cooling techniques
(eg, ice water immersion), fluid resuscitation, and treatment of end-organ dysfunction.
(Choice A) Anticholinergic toxicity can cause confusion, tachycardia, and temperature elevations; however,
severe hyperthermia ≥40 C (104 F) is unexpected. Furthermore, most patients have other signs of
anticholinergic toxicity (eg, pupillary dilation, flushing).
(Choice C) Malignant hyperthermia affects genetically susceptible individuals during anesthesia involving
agents like halothane and succinylcholine. Diffuse muscle rigidity is also expected.
(Choice D) Meningococcal meningitis can cause fever and mental status changes and is associated with
DIC. However, it would be expected to cause neck stiffness and petechial rash. Furthermore, it would be
unlikely to cause such high fevers or develop so rapidly in a patient who was in his normal state of health this
morning.
(Choice E) Salicylate toxicity causes fever and mental status changes; however, patients typically present with
nausea and vomiting. Furthermore, significant hyperpnea is typically seen due to activation of the medullary
respiratory center.
Educational objective:
Heat stroke is characterized by core temperature ≥40 C (104 F) with CNS dysfunction (eg, altered mental
status). It occurs most commonly in those exposed to hot/humid environments while performing extreme
activities. Complications include rhabdomyolysis, disseminated intravascular coagulation, and end-organ
dysfunction.

8.
This item has associated media that may require the use of headphones. Please ensure your system/speaker
volume is set to an audible level.
A 72-year-old man is hospitalized for a right femoral fracture following a motor vehicle collision. Six days after
undergoing surgical repair, he is found to be lethargic. The patient has hypertension and osteoarthritis. His
medications include hydrochlorothiazide, lisinopril, and naproxen, which were continued in the hospital. On
examination, the patient is drowsy but awakens briefly when addressed by name. Temperature is 36.7 C (98
F), blood pressure is 144/76 mm Hg, pulse is 88/min, and respirations are 16/min. Pulse oximetry is 95% on
room air. Multiple contusions are present in the anterior abdominal wall and thighs. Physical examination
findings are shown in the video clip. Laboratory results are as follows:
Hemoglobin 8.4 g/dL
Blood urea nitrogen 78 mg/dL
Albumin 3.8 g/dL
Total bilirubin 0.4 mg/dL
Aspartate aminotransferase 112 U/L
Alanine aminotransferase 42 U/L
Creatine kinase, serum 32,000 U/L
Which of the following is the most appropriate next step in management of this patient?
Media 1

(QID: 10776)

imedicaldoctor.net 12
UWORLD USMLE STEP 2 QBANK - MEDICINE - NERVOUS SYSTEM (MISCELLANEOUS) - September 2024

A. Blood transfusion B. Hemodialysis


C. Intravenous lorazepam D. Intravenous morphine
E. Lactulose
F. Stress test

Correct Answer is B (47% answered correctly)


Indications for urgent dialysis (AEIOU)
• Metabolic acidosis
Acidosis
o pH <7.1 refractory to medical therapy

• Symptomatic hyperkalemia
o ECG changes or ventricular arrhythmias
Electrolyte abnormalities
• Severe hyperkalemia
o Potassium >6.5 mEq/L refractory to medical therapy

• Toxic alcohols (methanol, ethylene glycol)


• Salicylate
Ingestion
• Lithium
• Sodium valproate, carbamazepine
Overload • Volume overload refractory to diuretics
• Symptomatic:
o Encephalopathy
Uremia
o Pericarditis
o Bleeding
The video clip demonstrates asterixis, a bilateral, nonrhythmic, alternate flexion and extension movement at the
wrist (flapping) that occurs when the wrist is extended with the arms outstretched. Although classically
associated with hepatic encephalopathy (HE), asterixis can also be seen with uremic encephalopathy (due to
acute or chronic renal failure) and CO2 retention. It likely develops due to interruption of the neural pathways
that sustain muscle contraction.
This patient has numerous factors that together can cause a severe degree of acute kidney injury. These
include rhabdomyolysis from trauma (elevated creatine kinase), use of nonsteroidal anti-inflammatory drugs (eg,
naproxen) and diuretics (eg, hydrochlorothiazide), loss of glomerular autoregulation with lisinopril, and
anemia. Reduced renal clearance of uremic toxins leads to high levels of blood urea nitrogen (BUN) and
symptoms of lethargy and somnolence. The diagnosis of uremia is based on clinical symptoms/signs and not
on an absolute BUN level. Symptoms typically appear at a BUN level of >100 mg/dL but can develop at lower
levels. Uremic encephalopathy is an indication for urgent hemodialysis.
(Choice A) This patient has anemia but does not need a blood transfusion because his hemoglobin is >8 g/dL,
he is asymptomatic, and he does not have ongoing bleeding.
(Choice C) Intravenous lorazepam can be used to treat an ongoing seizure.
(Choice D) Intravenous morphine is an opioid analgesic that may worsen this patient's drowsiness and
lethargy. It may occasionally cause seizures or myoclonus.
(Choice E) Lactulose is used for the management of HE in decompensated cirrhosis. Although asterixis may
be seen in HE, this patient has normal bilirubin and albumin levels, suggesting normal liver synthetic
function. His mildly increased aspartate aminotransferase level is likely from skeletal muscle injury rather than
liver injury.

imedicaldoctor.net 13
UWORLD USMLE STEP 2 QBANK - MEDICINE - NERVOUS SYSTEM (MISCELLANEOUS) - September 2024

(Choice F) This patient's high creatine kinase level is due to rhabdomyolysis associated with the motor vehicle
collision and is not typically seen in acute coronary syndrome.
Educational objective:
Asterixis is a flapping movement of the hands that occurs when the wrist is extended with the arms
outstretched. Common causes include hepatic encephalopathy, uremic encephalopathy, and
hypercapnia. Treating the underlying cause will improve neurological status and resolve asterixis.

9. A 75-year-old woman is evaluated for agitation and confusion while hospitalized. She was admitted 4 days
ago due to fever, productive cough, and shortness of breath. The patient was diagnosed with community-
acquired pneumonia and her condition has improved with antibiotics. However, for the past 2 days, she has had
episodes of confusion and agitation, especially during the evening and night. The patient's medical history
includes hypertension, osteoarthritis, and mild dementia. She lives with her husband at a senior living facility
and is able to perform her daily activities with minimal assistance. Her temperature is 37.0 C (98.6 F), blood
pressure is 130/80 mm Hg, and pulse is 84/min. Pulse oximetry is 98% on room air. The patient recognizes her
husband at the bedside but is not oriented to time or location. During the examination, she repeatedly tries to
climb out of bed but calms when reoriented. The patient follows simple instructions and has no focal weakness
or sensory loss. She has moist mucous membranes and crackles in the right upper lung. Laboratory testing
shows an improving leukocytosis and normal electrolytes. Which of the following would be most helpful in
managing this patient's current condition? (QID: 12248)
A. Encourage bed rest to minimize falls B. Initiate low-dose benzodiazepine therapy
C. Limit the patient's interactions with family D. Provide sensory overstimulation to keep the
members patient attentive
E. Reduce nighttime noise and disturbances
F. Use soft bilateral physical restraints

Correct Answer is E (80% answered correctly)


Management of hospital delirium
• Reduce noise, improve room lighting, open window blinds during the day & avoid frequent room
changes
• Constant observation by a familiar person at the bedside, preferably a family member
• Nonpharmacologic sleep aids for insomnia
• Early mobilization & minimal use of physical restraints
• Visual & hearing aids when appropriate
• Early volume repletion for dehydrated patients
• Adequate pain control
• Aggressive chronic disease management (eg, diabetes mellitus, COPD)
• Reduce polypharmacy
• Monitor & treat for metabolic disturbances, infections & drug toxicity
COPD = chronic obstructive pulmonary disease.

imedicaldoctor.net 14
UWORLD USMLE STEP 2 QBANK - MEDICINE - NERVOUS SYSTEM (MISCELLANEOUS) - September 2024

This hospitalized patient with acute-onset agitation and confusion associated with disorientation and increased
motor activity (eg, trying to climb out of bed frequently), especially in the evening and night, likely has
delirium. This condition can be seen in any hospitalized patient but most often occurs in the elderly and those
with underlying dementia or previous stroke (likely due to reduced cognitive reserves). It is thought to be
related to increased oxidative stress, neuroinflammatory mediators, and impaired neurotransmitter
functioning. It occurs relatively quickly (over hours to days) and manifests with vacillating severity (better in
morning, worse at night) and changes in level of consciousness. Precipitating factors are listed in the
accompanying table.
Delirium is reversible in the majority of cases. Management includes identifying and treating the underlying
inciting factor (eg, pneumonia with antibiotics) as well as introducing nonpharmacological measures such as
reduction of nighttime noise and disturbances, frequent verbal orientation, reassurance, interactions with family
members, or the presence of a trained sitter at the bedside.
(Choices A and F) Using soft bilateral physical restraints and encouraging bed rest in an attempt to minimize
falls can worsen agitation and result in impaired mobility and pressure ulcers. In general, these measures are
used only when nonpharmacological measures are ineffective and patients remain a significant risk to
themselves or others despite the use of medications (eg, antipsychotics).
(Choice B) Benzodiazepines (eg, lorazepam) are medications with a rapid onset of action that can be used to
control agitation and combative behavior. However, they can have a paradoxical effect in the elderly by causing
disinhibition, thereby worsening delirium.
(Choice C) Limiting a patient's interactions with family members can worsen delirium by disorienting them
further. Most patients have improved outcomes by interacting with familiar people.
(Choice D) Providing sensory overstimulation to keep the patient attentive can worsen agitation and lead to
worsening confusion by not allowing for proper rest.
Educational objective:
Delirium is a reversible disorder characterized by agitation and confusion, most frequently seen in the elderly or
those with underlying dementia. Determining the underlying cause, avoiding unnecessary medications, treating
infections and metabolic disturbances, and encouraging regular activity during the day while minimizing
disturbances at night can help patients recover more quickly.

10. A 27-year-old woman comes to the office for follow-up. The patient was last seen 2 months ago due to
generalized musculoskeletal pain and several months of fatigue. She was diagnosed with fibromyalgia and
prescribed a low-impact exercise program and pharmacotherapy. Her pain and fatigue have improved, but she
now notes intermittent episodes of "dizziness." Most of the episodes are short-lived, but she occasionally needs
to brace herself against a wall or sit until the dizziness resolves. She also notes worsened lethargy and frequent
dry mouth. Other medical history includes migraines and irritable bowel syndrome. The patient currently takes
daily amitriptyline for fibromyalgia, occasional acetaminophen for muscular pains, and polyethylene glycol as
needed for constipation. Temperature is 36.8 C (98.2 F), blood pressure is 110/70 mm Hg, pulse is 70/min, and
respirations are 14/min. BMI is 21 kg/m2. Mucous membranes are pink and moist, and there is no jugular venous
distension. Cardiopulmonary auscultation is normal. Neurologic examination demonstrates intact cranial nerves
and normal muscle strength in all extremities. Which of the following would be most helpful in determining the
cause of this patient's dizziness? (QID: 12302)
A. Carotid sinus massage B. Dix-Hallpike testing
C. Orthostatic blood pressure measurements D. Romberg testing with eyes closed
E. Tandem gait testing

Correct Answer is C (76% answered correctly)


This patient with fibromyalgia was recently initiated on amitriptyline. Amitriptyline exerts its therapeutic effects
by inhibiting the reuptake of norepinephrine and serotonin, but its use is often limited due to interaction with other
receptors, including:

imedicaldoctor.net 15
UWORLD USMLE STEP 2 QBANK - MEDICINE - NERVOUS SYSTEM (MISCELLANEOUS) - September 2024

• Muscarinic receptors, leading to anticholinergic symptoms (eg, dry mouth, constipation, urinary
retention)
• Histamine receptors, leading to lethargy
• Alpha-adrenergic receptors, leading to orthostatic hypotension
These side effects often limit treatment and result in medication discontinuation.
Orthostatic hypotension is particularly common, even at low-dose therapeutic levels of
amitriptyline. Manifestations may be subtle and typically include transient episodes described as "dizziness" or
lightheadedness with sudden postural change, and patients often have a history of bracing themselves against
a wall or sitting until the episode resolves. Diagnosis is made when orthostatic blood pressure measurement
reveals a systolic blood pressure decline ≥20 mm Hg or diastolic blood pressure decline ≥10 mm
Hg. Discontinuation of the medication usually resolves the symptoms.
(Choice A) Carotid sinus massage can be used to diagnose/treat tachyarrhythmias (eg, paroxysmal
supraventricular tachycardia). Although tachyarrhythmias can present with dizziness, most cases are also
associated with palpitations and shortness of breath.
(Choice B) The Dix-Hallpike maneuver can be used to diagnose benign paroxysmal positional vertigo. This
condition is characterized by short episodes of vertigo with or without nausea/vomiting due to specific head
movements (eg, rolling over in bed, looking up). Because this patient developed symptoms after the initiation of
amitriptyline, a medication side effect is more likely.
(Choice D) Sensory ataxia is often suspected when patients have a positive Romberg test. Sensory ataxia can
be associated with balance dysregulation but is not typically characterized by episodic dizziness.
(Choice E) Tandem gait testing can be used to assess for cerebellar ataxia, which generally causes
disequilibrium and balance issues. Patients do not typically have episodic dizziness that improves with leaning
against a wall or sitting.
Educational objective:
Amitriptyline is commonly used for depression, insomnia, and pain disorders but is frequently associated with
side effects, including orthostatic hypotension, lethargy, and anticholinergic symptoms (eg, dry mouth,
constipation, urinary retention). Discontinuing the medication usually resolves these symptoms.

11. A 52-year-old man is brought to the emergency department by his daughter due to a fall while climbing
stairs. He has mild pain in his right arm but no other obvious injuries. The patient has had frequent stumbling
and near-falls over the last 2 months. He has also had significant fatigue, chronic abdominal pain, constipation,
and recurrent headaches as well as a "pins-and-needles sensation" in his palms and soles. His daughter notes
he has been more forgetful recently. The patient has smoked a pack of cigarettes daily for 20 years and drinks
beer on weekends. A year ago, he started work as a janitorial custodian at a battery manufacturing plant and
finds his job moderately stressful. The patient has not had regular medical follow-ups. Blood pressure is 160/90
mm Hg, and pulse is 84/min. The abdomen is soft and nontender, and no masses are palpable. There is mild
tenderness over the right middle ulna with normal range of motion at the wrist and elbow. Neurological evaluation
shows reduced pinprick sensation bilaterally in hands and feet. There is weakness of adduction and abduction
of the fingers and of bilateral thigh and knee extensors. The patient has a wide-based gait and is unable to
tandem walk. Laboratory results are as follows:

imedicaldoctor.net 16
UWORLD USMLE STEP 2 QBANK - MEDICINE - NERVOUS SYSTEM (MISCELLANEOUS) - September 2024

Complete blood count


Hemoglobin 9.5 g/dL
Mean corpuscular volume 66 fL
Reticulocytes 4%
Platelets 180,000/mm3
Leukocytes 4,100/mm3

Serum chemistry
Blood urea nitrogen 14 mg/dL
Creatinine 1.9 mg/dL
Glucose 100 mg/dL
Aspartate aminotransferase 12 U/L
Alanine aminotransferase 24 U/L
Uric acid, serum 13 mg/dL
Which of the following is the most likely diagnosis for this patient's clinical presentation? (QID: 12395)

A. Alcohol use disorder B. Diabetes mellitus


C. Hypothyroidism D. Lead toxicity
E. Parkinson disease
F. Pernicious anemia

Correct Answer is D (88% answered correctly)


Lead poisoning in adults
Risk
• Occupational exposure (eg, lead paint, batteries, ammunition, construction)
factors
• Gastrointestinal (abdominal pain, constipation, anorexia)
Clinical
• Neurologic (cognitive deficits, peripheral neuropathy)
features
• Hematologic (anemia)
• Anemia
Laboratory • Elevated venous lead level
findings • Elevated serum zinc protoporphyrin level
• Basophilic stippling on peripheral smear
This patient likely has lead toxicity due to occupational exposure in a battery manufacturing plant. In addition
to his clinical presentation consisting of nonspecific general manifestations (eg, fatigue), other features include:
• Neuropsychiatric manifestations (eg, short-term memory loss, sensorimotor neuropathy, headaches,
ataxia)
• Gastrointestinal manifestations (eg, abdominal pain, constipation)
• Hypertension and possible nephrotoxicity (eg, elevated creatinine)
Inhibition of enzymes responsible for heme and RNA synthesis in both bone marrow and mature erythrocytes
can lead to microcytic anemia with basophilic stippling (there may be mild hemolysis due to lead effects on

imedicaldoctor.net 17
UWORLD USMLE STEP 2 QBANK - MEDICINE - NERVOUS SYSTEM (MISCELLANEOUS) - September 2024

erythrocyte membrane fragility), and impaired purine metabolism can result in hyperuricemia, both seen in this
patient.
Once absorbed (in adults, predominantly via the lungs), lead distributes throughout the blood, bones, and other
organs, affecting cell function throughout. Lead is predominantly stored in the skeleton and is released slowly,
potentially exerting its pathologic effects over decades. Diagnosis depends on establishing a history of lead
exposure accompanied by corroborating physical examination findings (eg, neurologic manifestations) and
elevated blood lead levels. Removal from the lead source and chelation therapy is the treatment for those with
symptoms and/or markedly elevated levels.
(Choice A) Alcohol use disorder is associated with peripheral neuropathy and occasionally ataxia and
hyperuricemia. However, it is also associated with macrocytic anemia and transaminase elevations, which are
not present in this patient.
(Choice B) When chronically uncontrolled, diabetes mellitus is associated with peripheral neuropathy (usually
with more sensory than motor deficits), nephropathy, and retinopathy. It is not associated with microcytic
anemia.
(Choice C) Hypothyroidism can cause sensory neuropathy and may be associated with motor weakness and
hyperuricemia. However, anemia due to hypothyroidism is generally either normocytic or macrocytic, and
without reticulocytosis.
(Choice E) Parkinson disease clinically presents with "pill-rolling" tremor, bradykinesia, rigidity, and postural
instability. Gait is shuffling rather than ataxic. Parkinson disease alone is associated with autonomic dysfunction
but not usually peripheral neuropathy, anemia, hyperuricemia, or nephropathy.
(Choice F) Pernicious anemia is an autoimmune disease targeting intrinsic factor and/or parietal cells, resulting
in B12 malabsorption. Subsequent B12 deficiency may result in megaloblastic (macrocytic) anemia, dementia,
and subacute combined degeneration with progressive weakness, spasticity, and incontinence.
Educational objective:
Chronic lead toxicity in adults presents with neuropsychiatric, gastrointestinal, and general symptoms including
peripheral sensorimotor neuropathy, fatigue, abdominal pain, and constipation. It can lead to hypertension,
nephropathy, hyperuricemia, and microcytic anemia with basophilic stippling seen on the peripheral
smear. Diagnosis hinges on a thorough history including potential sources of lead exposure.

imedicaldoctor.net 18
UWORLD USMLE STEP 2 QBANK - MEDICINE - NERVOUS SYSTEM (MISCELLANEOUS) - September 2024

12. A 30-year-old man comes to the office with fatigue and lethargy that has worsened over the last 2 weeks. He
has been forgetful lately and feels "exhausted" at the end of the day. The patient works as a contractor and is
currently renovating old houses for sale. He describes feeling "clumsy" and dropping things at work, as well as
tripping multiple times while climbing stairs. He also has abdominal pain that he attributes to constipation. The
patient drinks 1 or 2 beers each weekend and does not use tobacco or illicit drugs. His mother was diagnosed
with lupus and his older sister had thyroid surgery. Blood pressure is 120/80 mm Hg, and pulse is
76/min. Examination shows normal jugular venous pressure, no thyromegaly, clear lung fields, and normal first
and second heart sounds. The abdomen is soft and nontender. There is no hepatomegaly or
splenomegaly. There is weakness on dorsiflexion of bilateral wrists and feet. Upper and lower limb deep tendon
reflexes are 1+. Laboratory results are as follows:
Complete blood count
Hemoglobin 9.6 g/dL
Mean corpuscular volume 76 µm3
Reticulocytes 4%
Platelets 200,000/mm3
Leukocytes 4,100/mm3

Serum chemistry
Blood urea nitrogen 12 mg/dL
Creatinine 1.0 mg/dL
Uric acid, serum 11 mg/dL
Creatine phosphokinase levels are normal. Which of the following is most likely to improve this patient's
symptoms? (QID: 12396)
A. Allopurinol B. Calcium disodium EDTA
C. Cobalamin supplementation D. Levothyroxine
E. Plasmapheresis
F. Thiamine supplementation

Correct Answer is B (67% answered correctly)


Lead poisoning in adults
Risk
• Occupational exposure (eg, lead paint, batteries, ammunition, construction)
factors
• Gastrointestinal (abdominal pain, constipation, anorexia)
Clinical
• Neurologic (cognitive deficits, peripheral neuropathy)
features
• Hematologic (anemia)
• Anemia
Laboratory • Elevated venous lead level
findings • Elevated serum zinc protoporphyrin level
• Basophilic stippling on peripheral smear
This patient likely has acute lead toxicity from exposure to lead-based paint given his recent work renovating
old houses. His constellation of clinical manifestations, including nonspecific findings (eg, fatigue), consists of:

imedicaldoctor.net 19
UWORLD USMLE STEP 2 QBANK - MEDICINE - NERVOUS SYSTEM (MISCELLANEOUS) - September 2024

• Gastrointestinal manifestations: Constipation


• Neuropsychiatric manifestations: Sensorimotor neuropathy (eg, weakness with dorsiflexion), short-
term memory loss
• Hematologic manifestations: Microcytic anemia (often with basophilic stippling on peripheral smear),
hyperuricemia (due to impaired purine metabolism)
Chronic lead toxicity can cause hypertension, nephropathy, and more pronounced neuropsychiatric symptoms
(eg, psychosis).
Lead is absorbed predominantly via the lungs in adults. It is stored predominantly in the skeleton and is released
slowly, exerting its pathologic effects over decades. Diagnosis depends on establishing a history of lead
exposure accompanied by corroborating physical examination findings (eg, neurologic manifestations) and
elevated blood lead levels. The treatment for symptomatic lead poisoning is chelation therapy with an agent
such as calcium disodium EDTA. It is important to separate the patient from further lead exposure as the
chelating agent may otherwise increase lead absorption from the source.
(Choice A) Allopurinol is a xanthine oxidase inhibitor used as urate-lowering therapy to prevent recurrent
gout. This patient has hyperuricemia from lead-induced inhibition of purine synthesis, but no inflammatory
arthritis suggestive of gout. Urate-lowering therapy would be unlikely to improve the neuropsychiatric and other
symptoms related to lead toxicity.
(Choices C and F) Cobalamin (vitamin B12) deficiency (pernicious anemia, post-gastrectomy syndrome,
intestinal malabsorption) causes macrocytic (not microcytic) anemia and subacute combined degeneration (eg,
spasticity, incontinence, paraplegia). Thiamine (vitamin B1) deficiency, seen with alcohol abuse and white
rice/cereal-based diets, manifests as Wernicke-Korsakoff syndrome (eg, nystagmus, ophthalmoplegia, ataxia,
confabulation) or beriberi (eg, distal and symmetric peripheral neuropathy, cardiomyopathy).
(Choice D) Hypothyroidism can cause a sensory neuropathy and may be associated with motor weakness and
hyperuricemia. However, anemia due to hypothyroidism is generally either normocytic or macrocytic, without
reticulocytosis.
(Choice E) Plasmapheresis improves outcomes in patients with severe Guillain-Barré syndrome. Although the
syndrome presents with progressive motor neuropathy and decreased/absent deep-tendon reflexes, the sensory
and cognitive deficits are usually milder than those in this patient, and there is no direct association with
hyperuricemia or microcytic anemia.
Educational objective:
Chronic lead toxicity presents in adults with neuropsychiatric, gastrointestinal, and general symptoms, including
peripheral sensorimotor neuropathy, fatigue, abdominal pain, and constipation. It can lead to hypertension,
nephropathy, hyperuricemia, and microcytic anemia with basophilic stippling seen on peripheral
smear. Treatment for symptomatic patients involves chelation therapy with an agent such as calcium disodium
EDTA.

imedicaldoctor.net 20
UWORLD USMLE STEP 2 QBANK - MEDICINE - NERVOUS SYSTEM (MISCELLANEOUS) - September 2024

13. A 47-year-old woman is brought to the emergency department after being found unresponsive in her
garage. The patient has a history of chronic pain, depression, and prior suicide attempts. Temperature is 35 C
(95 F), blood pressure is 106/64 mm Hg, pulse is 108/min, and respirations are 22/min. Pulse oximetry is 96%
on room air. The patient withdraws all extremities to painful stimuli but does not follow commands. Bilateral
pupils are equal and reactive, and funduscopy shows no papilledema. Lung auscultation shows occasional
wheezes. No heart murmurs are present. The abdomen is soft and nontender with decreased bowel
sounds. There is no extremity edema. Laboratory results are as follows:
Sodium 144 mEq/L
Chloride 108 mEq/L
Bicarbonate 18 mEq/L
Creatinine 0.8 mg/dL
Glucose 120 mg/dL
Endotracheal intubation followed by mechanical ventilation and other supportive measures are begun. A brain
MRI obtained several days later is shown in the exhibit. Which of the following is the most likely cause of this
patient's current condition? (QID: 14195)

imedicaldoctor.net 21
UWORLD USMLE STEP 2 QBANK - MEDICINE - NERVOUS SYSTEM (MISCELLANEOUS) - September 2024

A. Acute salicylate toxicity B. Carbon monoxide poisoning


C. Opioid drug overdose D. Prolonged hypothermia
E. Toxic alcohol ingestion

Correct Answer is B (59% answered correctly)


Carbon monoxide poisoning
• Smoke inhalation
• Defective heating systems
Epidemiology
• Gas motors operating in poorly ventilated
areas
Mild-moderate
• Headache, confusion
• Malaise, dizziness, nausea
Manifestations
Severe
• Seizure, syncope, coma
• Myocardial ischemia, arrhythmias

• ABG: carboxyhemoglobin level


Diagnosis
• ECG ± cardiac enzymes
• High-flow 100% oxygen
Treatment
• Intubation/hyperbaric oxygen (severe)
ABG = arterial blood gas.
This patient most likely experienced acute carbon monoxide (CO) poisoning. This can result from inhalation

imedicaldoctor.net 22
UWORLD USMLE STEP 2 QBANK - MEDICINE - NERVOUS SYSTEM (MISCELLANEOUS) - September 2024

of car exhaust in a closed space (eg, a garage), a situation often intentionally created in a suicide attempt.
CO causes toxicity by impairing oxygen (O2) delivery and usage in 3 ways:
• CO binds to hemoglobin with greater affinity than O2, causing a large reduction in O2-carrying capacity.
• CO triggers a left shift in the hemoglobin dissociation curve, decreasing O2 unloading in the tissues.
• CO disrupts oxidative phosphorylation in mitochondria.
Patients with CO poisoning typically develop clinical manifestations of cerebral hypoxia, including headache,
dizziness, and confusion. Severe intoxication can cause myocardial ischemia, seizure, coma, and death. Vital
signs are often largely unremarkable. Pulse oximetry is usually normal, because the oximeter cannot
differentiate between oxyhemoglobin and carboxyhemoglobin. Laboratory results can show an anion gap
metabolic acidosis (AGMA) due to lactic acidosis from peripheral tissue hypoxia. Permanent hypoxic brain
injury can occur, as evidenced in this patient's MRI showing bilateral hyperintensity of the globus pallidus, an
area highly sensitive to hypoxic conditions.
(Choice A) Acute salicylate toxicity typically causes AGMA as well as tachypnea (leading to a primary
respiratory alkalosis and mixed acid-base disorder); however, hypoxic findings on brain MRI are not expected.
(Choice C) Opioid overdose causes respiratory depression and hypoventilation that may lead to hypoxic brain
injury. However, this patient's elevated respiratory rate, normal O2 saturation on room air, and reactive (rather
than pinpoint) pupils are not consistent with opioid overdose.
(Choice D) Severe hypothermia (ie, body temperature <28 C) is typically required to cause loss of
consciousness. Bradycardia and a reduced respiratory rate, which are not present in this patient, are expected
with such a degree of hypothermia.
(Choice E) Methanol or ethylene glycol intoxication can cause unresponsiveness and AGMA; however, these
diagnoses are made less likely by absence of papilledema and normal renal function, respectively.
Educational objective:
Acute carbon monoxide poisoning can occur due to inhalation of car exhaust in a closed space, often performed
intentionally in a suicide attempt. Toxicity results from impaired delivery and usage of oxygen, leading to clinical
manifestations of cerebral hypoxia (eg, headache, confusion, seizure, coma). Laboratory results may
demonstrate lactic acidosis, and permanent hypoxic brain injury can occur.

imedicaldoctor.net 23
UWORLD USMLE STEP 2 QBANK - MEDICINE - NERVOUS SYSTEM (MISCELLANEOUS) - September 2024

14. A 38-year-old hospitalized woman is evaluated for new-onset confusion. The patient has a prolonged history
of Crohn disease and was admitted 2 days ago due to worsening abdominal pain, nausea, and watery
diarrhea. She tested positive for Clostridioides (formerly Clostridium) difficile infection and is receiving oral
vancomycin and intravenous fluids. Her gastrointestinal symptoms have been improving, but since yesterday
the patient has been incoherent, disoriented, and unsteady. She has no other medical conditions but has had
small bowel resections due to stricture and fistula in the past and has lost 10 kg (22 lbs) over the past 6
months. The patient does not use tobacco, alcohol, or illicit drugs. Temperature is 36.6 C (98 F), blood pressure
is 120/70 mm Hg, and pulse is 92/min with no orthostatic changes. BMI is 19.5 kg/m2. On examination, she is
not oriented to time or place. Pupils are equal and reactive to light. Abduction of the right eye is limited and
elicits bilateral horizontal nystagmus. Motor strength and deep tendon reflexes are normal throughout. Finger-
to-nose and heel-to-shin testing are normal, but the gait is wide based. Which of the following is the most likely
cause of this patient's neurologic symptoms? (QID: 14369)
A. Immune-mediated demyelination B. Medication-induced delirium
C. Microbial meningoencephalitis D. Nutritional vitamin deficiency
E. Vertebrobasilar arterial occlusion

Correct Answer is D (71% answered correctly)

imedicaldoctor.net 24
UWORLD USMLE STEP 2 QBANK - MEDICINE - NERVOUS SYSTEM (MISCELLANEOUS) - September 2024

Wernicke encephalopathy
• Chronic alcohol use (most common)
Associated
• Malnutrition (eg, anorexia nervosa)
conditions
• Hyperemesis gravidarum
Pathophysiology • Thiamine deficiency
• Encephalopathy
• Oculomotor dysfunction (eg, horizontal nystagmus & bilateral abducens
Clinical features
palsy)
• Postural & gait ataxia
Treatment • Intravenous thiamine followed by glucose infusion
This patient with Crohn disease had multiple small bowel resections complicated by significant weight loss, a
finding that suggests malnutrition. She developed altered mental status, a lateral-gaze palsy (eg, limited eye
abduction) with horizontal nystagmus, and a wide-based gait after receiving intravenous fluids, which raises
concern for Wernicke encephalopathy (WE).
WE is a neurologic complication of thiamine deficiency; it classically manifests as the triad of ataxia,
encephalopathy (eg, lethargy, disorientation), and oculomotor dysfunction (eg, nystagmus, gaze palsies). It
is usually associated with long-standing heavy alcohol use; however, WE can be associated with any form of
chronic malnourishment (eg, short-gut syndrome, anorexia).
Iatrogenic WE can be precipitated in malnourished patients by the administration of dextrose (which this patient
likely received as a maintenance fluid), which depletes the last remaining stores of thiamine, a vitamin cofactor
for enzymes involved in glucose metabolism. There are no diagnostic laboratory studies to confirm WE;
therefore, intravenous thiamine supplementation should be promptly administered if the diagnosis is
suspected. Korsakoff syndrome, a late-stage complication of WE, is characterized by significant retrograde and
anterograde amnesia, often with confabulation. Up to 80% of patients with alcohol use disorder who have WE
develop Korsakoff syndrome; however, it occurs less frequently in WE patients without alcohol use disorder.
(Choice A) Immune-mediated demyelination occurs in a variety of diseases, including Guillain-Barré syndrome
(GBS) and multiple sclerosis. GBS typically causes a rapidly progressive, ascending paralysis with hyporeflexia,
and multiple sclerosis causes neurologic deficits (eg, diplopia, focal weakness, bowel/bladder dysfunction)
disseminated in time and space. Altered mentation is rare and occurs subacutely with severe disease.
(Choice B) Medication-induced delirium is common with anticholinergics, benzodiazepines, or opiate
medications, but is not expected with vancomycin. Although delirium is associated with confusion and
disorientation, focal neurologic deficits are unexpected.
(Choice C) Bacterial or fungal meningoencephalitis can cause focal neurologic deficits and altered mentation,
but they are typically associated with fever, headache, and meningismus.
(Choice E) Symptoms of vertebrobasilar artery occlusions are dependent on the area involved but often cause
ataxia or altered mentation. However, diplopia, hemiparesis/paraparesis, or Wallenberg syndrome (loss of
pain/temperature in the ipsilateral face and contralateral body) are common. In addition, cerebellar examinations
(eg, finger-to-nose; heel to shin) are typically positive.
Educational objective:
Wernicke encephalopathy is a neurologic complication of thiamine deficiency; it classically manifests as the triad
of ataxia, encephalopathy, and oculomotor dysfunction. It is usually associated with long-standing heavy alcohol
use; however, it may be caused by any disorder that causes chronic malnourishment (eg, short-gut
syndrome). Treatment is with intravenous thiamine.

15. A 68-year-old woman is brought to the emergency department due to slurring of speech. The patient's family
reports that the patient has been clumsy and incoherent over the past several days, and today her speech

imedicaldoctor.net 25
UWORLD USMLE STEP 2 QBANK - MEDICINE - NERVOUS SYSTEM (MISCELLANEOUS) - September 2024

became slurred. She has also had poor appetite and nausea. The patient has a history of hypertension, type 2
diabetes mellitus, and bipolar disorder. Medications include amlodipine, metformin, and lithium. Physical
examination shows no facial asymmetry. Muscle strength is diffusely decreased, and gait is
unsteady. Laboratory results are as follows:
Complete blood count
Hemoglobin 13.1 g/dL
Platelets 320,000/mm3
Leukocytes 8,200/mm3

Serum chemistry
Sodium 136 mEq/L
Potassium 3.8 mEq/L
Blood urea nitrogen 30 mg/dL
Creatinine 1.2 mg/dL
Glucose 120 mg/dL
Magnesium 2.0 mg/dL
A noncontrast CT scan of the brain shows no acute ischemia or hemorrhage. The patient is admitted for
observation and further evaluation. In the hospital, she has frequent episodes of bradycardia and sinus
pauses. Which of the following is the best next step in the management of this patient? (QID: 16272)
A. Antinuclear antibody test B. Echocardiography
C. MR imaging of the brain D. Rapid plasma reagin test
E. Serum lithium level test

Correct Answer is E (76% answered correctly)

imedicaldoctor.net 26
UWORLD USMLE STEP 2 QBANK - MEDICINE - NERVOUS SYSTEM (MISCELLANEOUS) - September 2024

Lithium toxicity
Acute toxicity
• Intentional overdose
Chronic toxicity
Etiology
• Decreased renal perfusion (↓ lithium clearance)
o Dehydration
o Thiazide diuretics, NSAIDs, ACE inhibitors

Acute toxicity
• Gastrointestinal: nausea, vomiting, diarrhea
• Late neurologic sequelae
Clinical features
Chronic toxicity (neurologic)
• Lethargy, confusion, agitation
• Ataxia, tremor/fasciculations, seizure

• Intravenous hydration
Treatment
• Hemodialysis (severe cases)
NSAID = nonsteroidal anti-inflammatory drug.
This patient likely has chronic lithium toxicity. Lithium has a narrow therapeutic window, and chronic toxicity
most commonly occurs in patients on prescribed lithium therapy who experience a decrease in lithium
clearance rate. Lithium is primarily renally cleared in a similar manner to sodium; therefore, any process that
reduces renal perfusion increases lithium reabsorption (decreases clearance). Dehydration, evidenced in this
patient by a blood urea nitrogen/creatinine ratio >20, is a common cause of toxicity. The elderly are especially
at risk for toxicity due to a relatively low baseline glomerular filtration rate and volume of distribution (ie, low
muscle mass), both of which further narrow the therapeutic window.
Patients with chronic lithium toxicity typically develop neurologic symptoms including lethargy, confusion,
slurred speech, tremor, ataxia, and seizures. Cardiac abnormalities can also occur, mainly in the form of QT
interval prolongation and bradycardia. The diagnosis is made by an elevated serum lithium level (eg, >1.5
mEq/L) in the setting of appropriate signs and symptoms. Treatment is with hydration initially; in severe cases,
hemodialysis is indicated.
(Choice A) Antinuclear antibody testing is useful in the initial evaluation for systemic lupus erythematosus
(SLE), which can rarely cause CNS vasculitis leading to confusion and other neurologic deficits. However, new-
onset SLE is unlikely in this patient of relatively advanced age with no other apparent disease manifestations
(eg, skin rash, cytopenia).
(Choice B) Echocardiography is useful in diagnosing endocarditis, which can cause conduction abnormalities
and stroke (due to embolization); however, endocarditis is less likely in the absence of leukocytosis.
(Choice C) In the early stages of acute stroke (eg, within 6-12 hr of symptom onset) MR imaging of the brain
may detect stroke that is missed on CT scan. However, this patient has been having neurologic symptoms for
several days; a stroke, if responsible, should be evident on CT scan.
(Choice D) Rapid plasma reagin (RPR) testing is useful in diagnosing syphilis. Neurologic involvement in
tertiary syphilis can manifest with confusion and ataxia; however, the cardiac manifestations of tertiary syphilis
typically involve the aortic root rather than the conduction system. Lithium toxicity is more likely in this patient
receiving chronic lithium therapy who has laboratory evidence of dehydration.
Educational objective:
Chronic lithium toxicity most commonly results from reduced lithium clearance due to decreased renal perfusion
(eg, dehydration). Patients typically develop neurologic symptoms including lethargy, confusion, ataxia, and
tremor. Cardiac abnormalities (eg, QT interval prolongation, bradycardia) may also be present.

imedicaldoctor.net 27
UWORLD USMLE STEP 2 QBANK - MEDICINE - NERVOUS SYSTEM (MISCELLANEOUS) - September 2024

16. A 55-year-old man is brought to the emergency department by his wife due to sudden-onset confusion. The
couple lives on the coast, and the patient was on his boat this morning performing recreational diving with 2 of
his friends. Shortly after returning home, the patient reported shortness of breath, as well as tingling and
weakness of the left arm. He attempted to walk to a chair but stumbled and fell. His speech then began to slur,
and he could not answer questions appropriately. Medical history includes hypertension and diet-controlled type
2 diabetes mellitus. Temperature is 37.2 C (99 F), blood pressure is 102/68 mm Hg, and pulse is
104/min. Physical examination shows moderate respiratory distress. Heart sounds are normal with no
murmurs. There are faint lung crackles bilaterally. Neurologic examination reveals reduced muscle strength in
the left arm and no other focal findings. Mottling of the skin of the hands and feet is also noted. Which of the
following is the most likely cause of this patient's presentation? (QID: 17007)
A. Carbon monoxide poisoning B. Cerebral artery plaque rupture
C. Cocaine intoxication D. Severe hyponatremia
E. Vascular air embolus

Correct Answer is E (86% answered correctly)


This patient is most likely experiencing the effects of vascular air embolism. Deep underwater diving (ie, scuba
diving) is associated with a risk of decompression sickness ("the bends"), which can result from the formation
of nitrogen gas bubbles within the body when a diver ascends too rapidly from the high ambient pressure of
the deep water to the lower ambient pressure at the surface. Symptoms usually begin within 12 hours of
surfacing.
Small air bubbles in the venous bloodstream can lodge in the capillaries of the skin to cause pruritus or mottling
and cyanosis of the extremities. The air bubbles can also lodge in the pulmonary capillaries to cause
respiratory distress and localized ischemia, with resulting pulmonary edema. A relatively large volume of
coalesced air (eg, 50 mL) can lodge in the right ventricular outflow tract and cause obstructive shock.
Air can also pass into the arterial circulation by overwhelming the pulmonary capillary filtering capacity or via a
right-to-left-shunt (eg, patent foramen ovale). Small air bubbles in the arterial circulation can travel to the brain
to cause confusion, gait ataxia, and dysarthria. A small volume (eg, 1-2 mL) of coalesced air can cause
localized stroke (eg, left arm weakness) or myocardial infarction.
Emergency treatment involves intravenous hydration, Trendelenburg positioning, and the immediate
administration of 100% oxygen to facilitate absorption of nitrogen gas from the bloodstream. Optimal
management is with hyperbaric oxygen therapy.
(Choice A) Carbon monoxide poisoning most often occurs due to exposure to smoke or faulty heating systems
in a closed space and usually presents with headache, nausea, dizziness, and confusion. Focal neurologic
deficits are not typical.
(Choice B) Cerebral artery plaque rupture is a common mechanism of stroke, and this patient has risk factors
for atherosclerotic disease (eg, diabetes mellitus, hypertension). However, plaque rupture would not explain this
patient's respiratory symptoms.
(Choice C) Cocaine intoxication can present with chest pain and respiratory distress and can increase the risk
of stroke; however, confusion, gait ataxia, and dysarthria are not typical.
(Choice D) Severe hyponatremia can occur following intense exertion with ingestion of hypotonic fluid. It
typically presents with confusion but would not explain this patient's respiratory distress or focal neurologic
findings.
Educational objective:
Vascular air embolism can occur due to decompression sickness following deep underwater diving. Venous air
embolism can cause skin cyanosis, respiratory distress, and obstructive shock, whereas arterial air embolism
can cause confusion, gait ataxia, dysarthria, and stroke.

imedicaldoctor.net 28
UWORLD USMLE STEP 2 QBANK - MEDICINE - NERVOUS SYSTEM (MISCELLANEOUS) - September 2024

17. A 34-year-old man is brought to the emergency department due to confusion. The patient recently lost his
job and moved in with his parents yesterday because he could no longer afford housing. Today, his mother
found him disoriented and unsteady. He has no known chronic medical conditions and takes no
medications. Temperature is 38.5 C (101.3 F), blood pressure is 164/90 mm Hg, pulse is 108/min, and
respirations are 22/min. The patient appears restless and is constantly picking at the bed linens, yelling, "Get
these bugs off me." The lungs are clear on auscultation, and cardiac examination is unremarkable with the
exception of regular tachycardia. The abdomen is soft and nontender. No extremity edema is
present. Laboratory results are as follows:
Complete blood count
Hemoglobin 14.8 g/dL
Mean corpuscular volume 104 µm3
Platelets 300,000/mm3
Leukocytes 11,000/mm3

Serum chemistry
Sodium 146 mEq/L
Potassium 3.1 mEq/L
Magnesium 1.8 mg/dL
Phosphorus 2.4 mg/dL
Creatinine 0.8 mg/dL
Which of the following would be most helpful to improve this patient's current condition? (QID: 18287)
A. Atypical antipsychotic agent B. Broad-spectrum antibiotic
C. Long-acting benzodiazepine D. Thyroid hormone replacement
E. Vitamin supplementation

Correct Answer is C (59% answered correctly)


This patient's acute confusion, disequilibrium, restlessness, and visual/tactile hallucinations raise strong
suspicion for alcohol withdrawal. Although ~14% of people in the United States have alcohol use disorder, it
is often masked until alcohol intake is abruptly reduced or stopped, as likely occurred when this patient changed
his living situation from living alone to living with his parents.
Alcohol is a strong CNS depressant that enhances GABA (inhibitory) signaling and reduces NMDA (excitatory)
signaling. Most patients with alcohol use disorder develop a new homeostasis whereby the depressant effects
of alcohol are required to counterbalance innate CNS excitatory signaling; therefore, reduced alcohol
consumption can result in rebound CNS overexcitation.
Manifestations usually begin 6-24 hours after alcohol cessation and include anxiety, agitation, tremor,
diaphoresis, and nausea. More severe cases are marked by ≥1 of the following:
• Alcohol hallucinations: visual hallucinations that often prominently feature insects or animals
• Delirium tremens: rapid-onset delirium, agitation, and extreme autonomic instability, including fever,
sinus tachycardia, hypertension, and diaphoresis
• Withdrawal seizures: general tonic-clonic seizures that often occur in rapid succession
Alcohol withdrawal is treated primarily with benzodiazepines. This class of medications activates the GABA A
receptor, which dampens excitatory signaling and somewhat replaces the effect of alcohol. Long-acting
benzodiazepenes (eg, chlordiazepoxide) help reduce the rate of recurrent withdrawal or seizures, leading to a

imedicaldoctor.net 29
UWORLD USMLE STEP 2 QBANK - MEDICINE - NERVOUS SYSTEM (MISCELLANEOUS) - September 2024

smoother clinical course. Significant electrolyte abnormalities and dehydration are also often present, so
electrolyte and fluid replacement is generally required.
(Choice A) Atypical antipsychotic agents treat a wide range of psychoses, including schizophrenia. Although
visual hallucinations are often present, this patient's concomitant autonomic instability, electrolyte abnormalities,
and macrocytosis make alcohol withdrawal more likely.
(Choice B) Acute bacterial infection with sepsis can cause delirium, fever, and sinus tachycardia. However,
visual hallucinations would be atypical. In addition, significant leukocytosis is generally present.
(Choice D) Hypothyroidism can cause delirium and hypertension, but fever, tachycardia, and visual
hallucinations would be atypical.
(Choice E) Thiamine deficiency is common in patients with alcohol use disorder. It can cause Wernicke
encephalopathy, which is associated with delirium, ataxia, and oculomotor dysfunction. However, autonomic
instability and visual hallucinations are atypical.
Educational objective:
Patients with alcohol use disorder are at risk for alcohol withdrawal with alcohol
reduction/cessation. Manifestations begin within 6-24 hours and include anxiety, agitation, tremor, diaphoresis,
and nausea. More severe cases are marked by delirium tremens (eg, autonomic instability, delirium),
hallucinations, and/or seizures. The primary treatment is with benzodiazepines, which dampen CNS excitation.

18. A 75-year-old man is brought to the office by his daughter due to increasing confusion over the past 3
weeks. His daughter noticed that he has had trouble writing, using a computer, and keeping track of his
calendar. The patient's wife died 4 weeks ago and since then, he has not been sleeping well and taking
doxylamine at bedtime has not helped much. Other medical conditions include chronic obstructive pulmonary
disease and hypertension. The patient takes amlodipine and tiotropium. Temperature is 36.1 C (97 F), blood
pressure is 140/90 mm Hg, pulse is 105/min, and respirations are 18/min. The patient is oriented to person but
not to time or place. Extraocular movements are normal. Lungs are clear to auscultation bilaterally. Reflexes
are 2+ and strength is normal. His gait is ataxic and his speech is dysarthric. Complete blood count, urinalysis,
and serum chemistries are normal. CT scan of the head reveals mild cortical atrophy but is otherwise
normal. Which of the following is the most appropriate next step in management? (QID: 20443)
A. Add donepezil therapy B. Add selective serotonin reuptake inhibitor
therapy
C. Discontinue doxylamine D. Electroencephalogram
E. Pre- and post-lumbar puncture gait
assessment
F. Transfer to an assisted living facility

Correct Answer is C (67% answered correctly)

imedicaldoctor.net 30
UWORLD USMLE STEP 2 QBANK - MEDICINE - NERVOUS SYSTEM (MISCELLANEOUS) - September 2024

Common agents with anticholinergic properties


Antihistamines • Diphenhydramine, cyproheptadine, doxylamine
• 1st generation: haloperidol
Psychotropics • 2nd generation: clozapine
• TCAs: amitriptyline, clomipramine, doxepin
Antiparkinson • Benztropine, trihexyphenidyl
• Ipratropium (bronchodilator)
• Oxybutynin, darifenacin (overactive bladder)
Antimuscarinics
• Dicyclomine, hyoscyamine (antispasmodics)
• Scopolamine (antiemetic)
Mydriatics • Atropine, cyclopentolate
• Muscarinic mushroom species
Plant-based • Jimson weed
• Nightshade species
TCAs = tricyclic antidepressants.
This patient's memory impairment, ataxia, and dysarthria that developed 3 weeks after initiation of doxylamine
(an anticholinergic sleeping medication) is most likely due to chronic anticholinergic toxicity caused by the
combined anticholinergic effects of doxylamine and tiotropium. Many medications with anticholinergic
properties can cause toxicity at therapeutic doses, particularly in combination; elderly psychiatric patients
taking multiple medications and young children are at highest risk. Removal of the anticholinergic medication
generally leads to symptom resolution; therefore, doxylamine should be discontinued in this patient.
Chronic anticholinergic toxicity often presents with subtle, nonspecific symptoms (eg, confusion, altered
mental status, dysarthria), and more classic findings typical of acute toxicity (eg, tachycardia, hyperthermia,
flushed dry skin, urinary retention) are often absent. Therefore, anticholinergic toxicity should always be
considered in patients taking multiple medications with nonspecific symptoms. For example, impaired memory,
gait disturbance, and confusion (common in the elderly) may be mistakenly attributed to aging or other disease
processes when the actual cause is cumulative anticholinergic effects.
(Choice A) Donepezil is a cholinesterase inhibitor used to treat Alzheimer dementia (AD), which has an insidious
onset. This patient's rapid onset of symptoms (3 weeks) is not consistent with AD.
(Choice B) Selective serotonin reuptake inhibitors are primarily used to treat depression. Although this patient's
cognitive impairment and sleep disturbance could be caused by depression, his abnormal neurologic
examination (gait ataxia and dysarthria) is inconsistent with a psychiatric diagnosis.
(Choice D) An electroencephalogram (EEG) can be used to characterize and stage dementia. The rapid onset
of symptoms in this patient is suggestive of an alternate diagnosis and an EEG would not be the next best step.
(Choice E) Pre- and post-lumbar puncture gait assessment is used to diagnose normal pressure hydrocephalus
(NPH), a condition characterized by a change in mental status, ataxia, and incontinence (not present in this
patient). A rapid time course is uncommon in NPH, and abnormal findings (eg, ventricular enlargement) would
likely be present on CT scan of the head.
(Choice F) Transfer to an assisted living facility could be appropriate in patients diagnosed with dementia and
at risk for falls. However, a thorough evaluation for reversible causes of a patient's symptoms should be
performed first.
Educational objective:
Patients treated with multiple anticholinergic medications (eg, elderly psychiatric patients) are at risk for chronic
anticholinergic toxicity, which often presents with subtle and nonspecific findings not typical of acute

imedicaldoctor.net 31
UWORLD USMLE STEP 2 QBANK - MEDICINE - NERVOUS SYSTEM (MISCELLANEOUS) - September 2024

anticholinergic toxicity. Removal of the anticholinergic medication leads to resolution of symptoms.

19. A 20-year-old college student comes to the office due to worsening abnormal movements. The symptoms
initially started in middle school when he would repeatedly blink and then shrug his shoulders. The patient
worked with a counselor who taught him techniques to suppress movements for periods of time, although these
did not completely resolve them. Over the past few years, the symptoms have gradually worsened. Episodes
now occur multiple times daily, particularly around highly stressful times at school. His grades have declined,
and he is embarrassed during class presentations. The patient has no history of serious illness and takes no
medications. He occasionally smokes marijuana on weekends. Family history is notable for the death of his
father at age 50 from a stroke. Vital signs are normal. During the examination, the patient is visibly anxious. He
repeatedly blinks, clears his throat, and shrugs his shoulders. The remainder of the physical examination shows
no abnormalities. Which of the following is the most appropriate treatment for this patient? (QID: 20778)
A. Antisense oligonucleotide therapy B. Copper chelation therapy
C. GABA agonist therapy D. Serotonin reuptake inhibition therapy
E. VMAT2 inhibitor therapy

Correct Answer is E (48% answered correctly)


Tourette syndrome
• Both multiple motor & ≥1 vocal tics (not necessarily
concurrent, >1 year)
o Motor: facial grimacing, blinking, head/neck
Clinical jerking, shoulder shrugging, tongue
features protrusion, sniffing
o Vocal: grunting, snorting, throat clearing,
barking, yelling, coprolalia (obscenities)
• Onset age <18
• Attention deficit hyperactivity disorder
Comorbidity
• Obsessive-compulsive disorder
• Education/watchful waiting if mild & nondisabling
• Behavioral therapy (habit reversal training)
• Antidopaminergic agents
Treatment
o Tetrabenazine (VMAT2 inhibitor)
o Antipsychotics (receptor blockers)
• Alpha-2 adrenergic receptor agonists
• Severity peaks age 10-12
Prognosis • Majority resolve or improve in adulthood; tics persist
in 1/3
This patient's presentation is most consistent with Tourette syndrome (TS). TS, a neurological disorder with
childhood onset, is characterized by a combination of vocal (eg, throat clearing) and motor (eg, blinking,
shoulder shrugging) tics. Tic severity typically peaks at age 10-12, and tics are exacerbated by anxiety, fatigue,
and psychosocial stress. When tics are severe and debilitating (eg, declining academic performance,
psychosocial dysfunction), as in this patient, treatment options include behavioral suppressive therapy (eg,
habit reversal training, substituting a voluntary movement incompatible with the tic) and pharmacological
interventions.

imedicaldoctor.net 32
UWORLD USMLE STEP 2 QBANK - MEDICINE - NERVOUS SYSTEM (MISCELLANEOUS) - September 2024

Pharmacotherapy of TS consists of:


• Antidopaminergic drugs: Vesicular monoamine transporter type 2 (VMAT2) inhibitors (eg,
tetrabenazine) function as dopamine depleters, whereas antipsychotic medications (eg, risperidone)
block dopamine postsynaptic receptors and carry risks of parkinsonism, tardive dyskinesia, and metabolic
effects. VMAT2 inhibitors are therefore increasingly preferred as initial treatment over antipsychotics.
• Alpha-2 adrenergic receptor agonists (eg, guanfacine, clonidine) can be considered in patients with
comorbid attention deficit hyperactivity disorder or behavioral symptoms.
(Choice A) Antisense oligonucleotide therapy is an investigational treatment for Huntington disease (HD), a
progressive neurodegenerative disorder characterized by choreiform movements and neuropsychiatric
symptoms, including dementia. However, it is an inherited condition (eg, family history of progressive motor and
cognitive decline) that is most common in midlife, with diagnosis prior to the age of 20 being rare. In addition,
this patient's partial response to behavioral suppressive therapy in middle school makes TS more likely than HD.
(Choice B) Copper chelation therapy is used in the treatment of Wilson disease, an autosomal recessive
disorder that presents with hepatic, neurologic, and psychiatric symptoms. Neurological manifestations
commonly include dysarthria, dystonia, ataxia, parkinsonism, and drooling, rather than the waxing and waning
tics seen in this patient. In addition, this patient has no hepatic impairment, making TS the more likely diagnosis.
(Choice C) The GABA agonist medication clonazepam (a benzodiazepine) is a first-line treatment option for
myoclonus; this patient's motor and vocal tics are consistent with TS rather than myoclonus.
(Choice D) Serotonergic drugs (eg, selective serotonin reuptake inhibitors) would not effectively target this
patient's tics. These medications may help treat patients with TS who have comorbid anxiety or obsessive-
compulsive disorder.
Educational objective:
Pharmacotherapy of Tourette syndrome consists of vesicular monoamine transporter type 2 (VMAT2) inhibitors,
antipsychotic medications, and alpha-2 adrenergic receptor agonists. VMAT2 inhibitors are increasingly
preferred as initial treatment over antipsychotics due to a more favorable adverse effect profile.

20. A 68-year-old woman is admitted to the hospital due to a femoral neck fracture after a traumatic fall. The
fracture is repaired, and her pain is treated with as-needed opioid medications. Over the course of her
hospitalization, the patient develops nocturnal disorientation and mild agitation. Her husband says she is not
like this at home. During morning rounds, the patient is pleasant and answers questions appropriately. Her
condition resolves over the next few days, and she is discharged to the rehabilitation facility on day 7. Because
of this patient's mental status changes, she is at increased risk for which of the following? (QID: 21328)
A. Acute mania B. Cognitive decline
C. Depression with psychotic features D. Nonconvulsive epilepsy
E. Opioid dependence

Correct Answer is B (70% answered correctly)

imedicaldoctor.net 33
UWORLD USMLE STEP 2 QBANK - MEDICINE - NERVOUS SYSTEM (MISCELLANEOUS) - September 2024

Common complications of delirium


• Disorientation, falls
• Immobility, pressure ulcers
Short term • Poor intake, dehydration
• Aspiration pneumonia
• Prolonged hospitalization
• Persistent delirium
Long term
• Nursing home placement
• Cognitive decline/dementia
Permanent
• Death: ~20% mortality at 6 months
This patient has fluctuating, nocturnal disorientation and agitation in the setting of a medical illness (eg, hip
fracture, surgery) consistent with delirium. Delirium is typically worse at night; however, fluctuation may allow
patients to seem lucid during the day, resulting in underdiagnosis. Hyperactivity (eg, agitation) is classic,
although hypoactive delirium (eg, lethargy, somnolence) is more common in the elderly (often misdiagnosed as
depression). In this patient, either uncontrolled postoperative pain or opioid use was the most likely cause of
her delirium.
Although the acute symptoms of delirium are usually transient and resolve with treatment of the underlying cause,
some patients experience persistent cognitive dysfunction. Delirium also accelerates the progression of
dementia, likely because it causes direct neurologic injury through similar mechanisms (eg, neuroinflammation,
decreased cerebral blood flow, impaired neurotransmitter function). Even in patients without a history of
dementia, a single episode of acute delirium increases the risk for further cognitive decline. In addition, delirium
can portend a downward spiral of functional loss, long-term facility placement, and death.
(Choice A) Acute mania can present with agitation similar to that in hyperactive delirium. However, expansive
symptoms (eg, elevated mood, grandiosity, pressured speech) are usually present, and fluctuations in
consciousness do not occur. Delirium does not increase the risk of mania.
(Choice C) Major depressive disorder can have psychotic features (eg, hallucinations, delusions) similar to
those of delirium. Delirium is distinguished by the acute onset of confusion/fluctuating consciousness, and it is
not associated with psychotic depression.
(Choice D) Nonconvulsive status epilepticus (NCSE) describes ongoing or intermittent seizures without
convulsions. Both NCSE and delirium are more frequent in the setting of critical illness and present with
fluctuating levels of consciousness. However, NCSE usually demonstrates additional symptoms (eg, catatonia,
automatisms, eye deviation), and delirium is not a risk factor.
(Choice E) Opioids are the primary treatment for postoperative pain. All postsurgical patients have an increased
risk for opioid dependence, especially those requiring opioids beyond 5 days. The risk of opioid dependence is
also increased by a previous history of substance abuse, chronic pain, or psychiatric disorder. The acute mental
status changes of delirium do not increase the risk of opioid dependence.
Educational objective:
Delirium is characterized by acute-onset, fluctuating consciousness, most frequently seen in elderly hospitalized
patients. It usually resolves with treatment. However, cognitive dysfunction can be persistent, and a single
episode of delirium increases the risk of further cognitive decline.

21. A 75-year-old man is brought to the hospital for a 2-day history of intermittent confusion and agitation. He
refused to allow his visiting nurse into his house for the first time. The patient was discharged from the hospital
a week ago after a lumbar laminectomy due to degenerative disc disease. His other medical conditions include
hypertension and chronic kidney disease. He takes lisinopril and acetaminophen/hydrocodone as needed for
pain. Temperature is 38.5 C (101.3 F), blood pressure is 112/70 mm Hg, pulse is 102/min, and respirations are
16/min. Pulse oximetry is 97% on room air. On mental status examination, the patient gets distracted and is

imedicaldoctor.net 34
UWORLD USMLE STEP 2 QBANK - MEDICINE - NERVOUS SYSTEM (MISCELLANEOUS) - September 2024

oriented to person but not time or place. Crackles are


present at both lung bases. Heart sounds are
normal. The abdomen is soft and nontender. The
remainder of the physical examination shows no
abnormalities. Laboratory studies obtained on arrival are
as follows:
Complete blood count
Hemoglobin 11 g/dL
Leukocytes 13,000/mm3

Serum chemistry
Sodium 136 mEq/L
Potassium 4.4 mEq/L
Bicarbonate 22 mEq/L
Blood urea nitrogen 30 mg/dL
Creatinine 1.9 mg/dL
Glucose 140 mg/dL

Urinalysis
Specific gravity 1.019
Leukocyte esterase negative
Nitrites negative
Bacteria none
White blood cells 1-2/hpf
Red blood cells 1-2/hpf
What is the best next step in the management of this patient? (QID: 21331)
A. Chest x-ray B. CT scan of the head
C. Electroencephalogram D. Lumbar puncture for spinal fluid evaluation
E. Renal ultrasound

Correct Answer is A (61% answered correctly)

This patient has delirium as determined by the Confusion Assessment Method (CAM), which requires:
• an acute (eg, 2-day), fluctuating (eg, intermittent) course
• inattention (eg, easily distractible)
• and either disorganized thinking (eg, confusion) or altered consciousness
The CAM is a validated screening tool that is both sensitive and specific for delirium. Risk factors for delirium in
this patient include age, postoperative status, and opioid medications.
Once recognized, delirium should be considered a medical emergency. Because delirium is often associated
with an underlying medical condition, initial evaluation includes a history and physical examination, medication
review, basic laboratory workup, and urinalysis to look for the most common causes. Abnormal findings help

imedicaldoctor.net 35
UWORLD USMLE STEP 2 QBANK - MEDICINE - NERVOUS SYSTEM (MISCELLANEOUS) - September 2024

guide further evaluation and treatment. This patient's fever, crackles, and leukocytosis suggest underlying
pneumonia, which warrants a chest x-ray.
However, delirium is often multifactorial; if there is inadequate treatment response or initial workup is negative,
extended investigation (eg, based on the patient's history) is warranted.
(Choices B and D) CT scan of the head is indicated in patients with delirium when history (eg, headache,
trauma, anticoagulation) or physical examination (eg, focal weakness, papilledema) suggests intracranial
pathology; it is not initially necessary when focal neurologic findings are absent. Lumbar puncture is typically
used to diagnose meningitis or encephalitis, which can cause fever and altered mental status and infrequently
complicate spinal surgery. However, this patient does not have meningismus, and his fever and crackles are
likely due to pneumonia. If initial workup is negative or treatment response is inadequate, neuroimaging and
lumbar puncture should be considered.
(Choice C) Electroencephalogram (EEG) can help identify occult seizures as the cause of altered mental
status. Patients with a concerning history (eg, stroke, trauma, critical illness) or inadequate response to
treatment of the underlying cause require EEG to rule out nonconvulsive seizures as the etiology of delirium.
(Choice E) Renal ultrasound is useful to exclude complications of genitourinary infection (eg, perinephric
abscess) and urinary tract obstruction accompanying acute kidney injury. This patient's findings do not suggest
infection (eg, negative urinalysis) or retention (eg, no bladder distension on abdominal examination). His
creatinine is likely elevated due to chronic kidney disease and dehydration.
Educational objective:
Delirium is an acute, fluctuating disturbance in attention, thinking, and consciousness. Abnormal findings on
initial evaluation guide further investigation (eg, chest x-ray due to fever, crackles, and leukocytosis) and
treatment. If initial evaluation is negative or there is inadequate response to treatment, extended investigation
is warranted.

22. A 65-year-old woman comes to the office due to memory impairment. The patient's memory difficulties
began 2 months ago, and she is concerned her symptoms have recently worsened. She often forgets parts of
recent conversations and has misplaced her car keys multiple times. The patient sleeps without issue and feels
well rested upon awakening. She is a retired accountant and recently moved across the country to care for her
parents, preparing meals for them and managing their finances. The patient regularly eats out at restaurants
and drinks a glass of wine 3-4 times a week. Medical history includes allergic rhinitis, for which she has been
taking over-the-counter medications. Temperature is 36.9 C (98.4 F), blood pressure is 130/82 mm Hg, and
pulse is 76/min. The patient is alert and oriented. Physical examination shows no abnormalities. Which of the
following is the most appropriate statement? (QID: 21721)
A. "A healthy lifestyle can help reduce further B. "I'm concerned you may have early signs of
memory decline; I recommend eating a Alzheimer disease; neuropsychological testing can
balanced diet and exercising at least 3-4 times help determine the extent of your symptoms."
a week."
C. "Sometimes medications can contribute to D. "Stress related to being a caregiver may be
the symptoms you are experiencing; let's causing memory issues; have you considered
review what you have been taking recently." hiring someone to help out with your parents?"
E. "Unfortunately, occasional forgetfulness is a
normal part of aging; cognitive strategies can
sometimes improve memory retention."

Correct Answer is C (56% answered correctly)


This patient has memory impairment that began recently and had a relatively abrupt onset, with no significant
functional decline and a normal physical examination. This presentation should prompt consideration of
reversible causes, including a careful review of medications. Many over-the-counter (OTC) allergy medications
(eg, antihistamines) have anticholinergic effects that can contribute to memory impairment, especially in older

imedicaldoctor.net 36
UWORLD USMLE STEP 2 QBANK - MEDICINE - NERVOUS SYSTEM (MISCELLANEOUS) - September 2024

patients. Memory impairment caused by medications with anticholinergic properties may be mistakenly
attributed to normal aging or the mild cognitive impairment characteristic of early dementia.
In general, many medications (both prescription and OTC), supplements, and substances may contribute to
memory impairment; therefore, obtaining a thorough social and medication history is an important step in
identifying modifiable causes of cognitive change. Physicians should prioritize discontinuing nonessential
medications that may be contributing to cognitive impairment and monitor for resolution of symptoms prior to
pursuing further workup or interventions.
(Choice A) Recommending a healthy lifestyle before investigating reversible causes of memory impairment (eg,
medications, substance use) would be inappropriate and would not address possible underlying contributors.
(Choice B) This statement is unnecessarily alarming and mistakenly assumes the patient's memory impairment
is due to Alzheimer disease. Alzheimer disease is typically associated with signs of functional impairment and
less likely to develop abruptly and progress quickly (eg, few weeks).
(Choices D and E) Although stress can cause memory impairment, it would be premature to attribute this
patient's memory impairment to stress or normal aging without first performing a careful review of her current
medications to identify potential offending agents. Normal aging is also less likely to cause abrupt, rapidly
progressive symptoms.
Educational objective:
Review of medications should be prioritized in evaluating new-onset or worsening cognitive
impairment. Medications with anticholinergic properties are strongly associated with risk for cognitive
impairment, particularly in the elderly.

23. A 91-year-old woman is brought to the emergency department by her son due to confusion, increasing
forgetfulness, and lethargy over the past several weeks. She normally ambulates with a walker but is now unable
to do so due to an unsteady gait. She has had no fevers, chills, cough, nausea, vomiting, or diarrhea. Medical
history is significant for hypertension, for which she takes lisinopril and chlorthalidone. Temperature is 36.9 C
(98.4 F), blood pressure is 128/88 mm Hg supine and 120/85 standing, and pulse is 76/min. BMI is 19.5
kg/m2. Physical examination shows moist mucous membranes. Cardiopulmonary examination is
unremarkable. The patient is oriented to name only and has poor word recall. Strength and sensation are
intact. Deep tendon reflexes are normal. Laboratory results are as follows:
Serum chemistry
Sodium 123 mEq/L
Potassium 3.4 mEq/L
Chloride 90 mEq/L
Bicarbonate 25 mEq/L
Blood urea nitrogen 23 mg/dL
Creatinine 1.1 mg/dL
Calcium 9.9 mg/dL
Glucose 114 mg/dL
Urinalysis is unremarkable. CT scan of the head reveals mild brain atrophy. Which of the following is the most
appropriate next step in management of this patient? (QID: 114914)
A. Obtain MRI of the brain B. Obtain serum vitamin B12 level
C. Perform lumbar puncture D. Stop chlorthalidone
E. Stop lisinopril

imedicaldoctor.net 37
UWORLD USMLE STEP 2 QBANK - MEDICINE - NERVOUS SYSTEM (MISCELLANEOUS) - September 2024

Correct Answer is D (71% answered correctly)


Thiazide-induced hyponatremia
• ↓ Water excretion (ADH-dependent & ADH-independent
Pathogenesis mechanisms)
• ↑ Water intake (natural tendency to polydipsia)
• Advanced age (mean age >75), female, low BMI
Risk factors & • Typically within few weeks of therapy or after acute event (eg,
presentation diarrhea)
• Patients often euvolemic with subtle symptoms (eg, confusion)
• Discontinue thiazide
Treatment
• Correct hyponatremia (eg, IV fluids)
ADH = antidiuretic hormone; IV = intravenous.
This elderly patient has nonspecific neurologic findings—a few weeks of encephalopathy, gait instability, and
poor recall—that may be attributed to a variety of conditions. However, her laboratory results show significant
hyponatremia, which, in the setting of chlorthalidone use, raises suspicion for thiazide-induced chronic
hyponatremia.
Chronic (>48 hr) hyponatremia manifestations range from subtle (eg, nausea, fatigue) to more pronounced (eg,
encephalopathy, gait disturbances, seizures). Thiazide-induced hyponatremia can occur any time after
initiation (eg, months later), particularly in elderly female patients with a low BMI. Patient factors (eg, age-related
reduced ability to dilute urine, genetic predisposition to water retention) likely contribute. Although some patients
have mild volume depletion, most are euvolemic (eg, moist mucous membranes), likely reflecting ADH-
independent mechanisms (eg, direct thiazide effects on water retention). Management includes permanent
thiazide discontinuation (with an alternate agent used for blood pressure control, if needed) and intravenous
fluid administration; hypertonic saline is typically reserved for severe cases (eg, seizures).
Other causes of encephalopathy, including infections, other metabolic disorders, and structural brain
abnormalities, should be evaluated but are unlikely in this patient.
• Structural brain disorders (eg, stroke, subdural hematoma) are unlikely given the lack of focal deficits and
unremarkable head CT scan (mild brain atrophy is normal in elderly patients). MRI of the brain (eg, for
evaluation of old/small stroke or other structural abnormalities) could be considered if this patient's
symptoms persist without a clear cause despite thiazide cessation and hyponatremia correction (Choice
A).
• Vitamin B12 deficiency and some endocrine disorders can lead to gait instability and
forgetfulness. However, vitamin B12 deficiency commonly causes paresthesia, neuropathy, and
weakness (vs this patient's intact strength, sensation, and reflexes) (Choice B). Although severe
hypothyroidism and adrenal insufficiency can also cause hyponatremia, normal reflexes and vital signs
make these unlikely.
• Infection should always be excluded as a cause of encephalopathy. It is unlikely in this patient given the
absence of fever, a normal urinalysis, and clear lungs; meningitis and encephalitis (requiring lumbar
puncture) would also typically cause fever and have an acute (eg, hours to days) presentation (Choice
C).
ACE inhibitors (eg, lisinopril) can occasionally lead to renal failure, which can cause encephalopathy. However,
this patient's renal function is intact, and hyponatremia would not be expected (Choice E).
Educational objective:
Chronic hyponatremia manifestations can range from subtle (eg, nausea, fatigue) to more pronounced (eg,
encephalopathy, gait disturbances, seizures). Thiazides (eg, chlorthalidone) can cause hyponatremia,
particularly in elderly women with a low BMI; patients are typically euvolemic. Initial management includes
thiazide cessation.

imedicaldoctor.net 38

You might also like

pFad - Phonifier reborn

Pfad - The Proxy pFad of © 2024 Garber Painting. All rights reserved.

Note: This service is not intended for secure transactions such as banking, social media, email, or purchasing. Use at your own risk. We assume no liability whatsoever for broken pages.


Alternative Proxies:

Alternative Proxy

pFad Proxy

pFad v3 Proxy

pFad v4 Proxy